Вы находитесь на странице: 1из 55

5MEDBOOK.

COM

ONLY $5

Amazon.com http://www.amazo...

Find your book you want Send us Book URL Receive Free 55 first pages
in any site within 5 hours

5MEDBOOK.COM
FIRST AID ™

Q&A FOR THE





USMLE


STEP 1


Third Edition

SENIOR EDITORS
Tao Le, MD, MHS James A. Feinstein, MD
Associate Clinical Professor Clinical Instructor, Section of General Pediatrics
Chief, Section of Allergy and Clinical Immunology The Children's Hospital Colorado
Department of Medicine Research Fellow
University of Louisville Primary Care Research Program
University of Colorado School of Medicine

EDITORS
Mark W. Ball, MD Kimberly Kallianos
Resident Harvard Medical School
The James Buchanan Brady Urological Institute Class of 2012
The Johns Hopkins Hospital
Cesar Raudel Padilla
Annie Dude, MD University of Rochester School of Medicine
Resident Class of 2012
Department of Obstetrics and Gynecology
Duke University Medical Center Lauren Rothkopf, MD
Masters in Public Health candidate
Rebecca L. Hoffman, MD Temple University College of Health Professions and
Resident Social Work
Department of General Surgery
Hospital of the University of Pennsylvania James Yeh, MD
Resident
Mark Robert Jensen Department of Medicine
University of Rochester School of Medicine Cambridge Hospital, Cambridge Health Alliance
Class of 2012

New York / Chicago / San Francisco / Lisbon / London / Madrid / Mexico City
Milan / New Delhi / San Juan / Seoul / Singapore / Sydney / Toronto

Copyright © 2012 by Tao Le. All rights reserved. Except as permitted under the United States Copyright Act of 1976, no part of this publication may be reproduced
or distributed in any form or by any means, or stored in a database or retrieval system, without the prior written permission of the publisher.

ISBN: 978-0-07-174596-3

MHID: 0-07-174596-3

The material in this eBook also appears in the print version of this title: ISBN: 978-0-07-174402-7,
MHID: 0-07-174402-9.

All trademarks are trademarks of their respective owners. Rather than put a trademark symbol after every occurrence of a trademarked name, we use names in an
editorial fashion only, and to the benefit of the trademark owner, with no intention of infringement of the trademark. Where such designations appear in this book,
they have been printed with initial caps.

McGraw-Hill eBooks are available at special quantity discounts to use as premiums and sales promotions, or for use in corporate training programs. To contact a
representative please e-mail us at bulksales@mcgraw-hill.com.

Notice

Medicine is an ever-changing science. As new research and clinical experience broaden our knowledge, changes in treatment and drug therapy are required. The
authors and the publisher of this work have checked with sources believed to be reliable in their efforts to provide information that is complete and generally in
accord with the standards accepted at the time of publication. However, in view of the possibility of human error or changes in medical sciences, neither the authors
nor the publisher nor any other party who has been involved in the preparation or publication of this work warrants that the information contained herein is in every
respect accurate or complete, and they disclaim all responsibility for any errors or omissions or for the results obtained from use of the information contained in
this work. Readers are encouraged to confirm the information contained herein with other sources. For example and in particular, readers are advised to check the
product information sheet included in the package of each drug they plan to administer to be certain that the information contained in this work is accurate and that
changes have not been made in the recommended dose or in the contraindications for administration. This recommendation is of particular
importance in connection with new or infrequently used drugs.

TERMS OF USE

This is a copyrighted work and The McGraw-Hill Companies, Inc. (“McGrawHill”) and its licensors reserve all rights in and to the work. Use of this work is subject
to these terms. Except as permitted under the Copyright Act of 1976 and the right to store and retrieve one copy of the work, you may not decompile, disassemble,
reverse engineer, reproduce, modify, create derivative works based upon, transmit, distribute, disseminate, sell, publish or sublicense the work or any part of it
without McGraw-Hill’s prior consent. You may use the work for your own noncommercial and personal use; any other use of the work is strictly prohibited. Your
right to use the work may be terminated if you fail to comply with these terms.

THE WORK IS PROVIDED “AS IS.” McGRAW-HILL AND ITS LICENSORS MAKE NO GUARANTEES OR WARRANTIES AS TO THE ACCURACY,
ADEQUACY OR COMPLETENESS OF OR RESULTS TO BE OBTAINED FROM USING THE WORK, INCLUDING ANY INFORMATION THAT CAN
BE ACCESSED THROUGH THE WORK VIA HYPERLINK OR OTHERWISE, AND EXPRESSLY DISCLAIM ANY WARRANTY, EXPRESS OR IMPLIED,
INCLUDING BUT NOT LIMITED TO IMPLIED WARRANTIES OF MERCHANTABILITY OR FITNESS FOR A PARTICULAR PURPOSE. McGraw-Hill
and its licensors do not warrant or guarantee that the functions contained in the work will meet your requirements or that its operation will be uninterrupted or
error free. Neither McGraw-Hill nor its licensors shall be liable to you or anyone else for any inaccuracy, error or omission, regardless of cause, in the work or for
any damages resulting therefrom. McGraw-Hill has no responsibility for the content of any information accessed through the work. Under no circumstances shall
McGraw-Hill and/or its licensors be liable for any indirect, incidental, special, punitive, consequential or similar damages that result from the use of or inability
to use the work, even if any of them has been advised of the possibility of such damages. This limitation of liability shall apply to any claim or cause whatsoever
whether such claim or cause arises in contract, tort or otherwise.
P O W E R E D BY usmte

FIRST AID
STEP 1 EXPRESS

FIRST AID

RESPIRATORY
PHYSIOLOGY
TAO LE, MD, MHS The official FIRST AID course
Assistant Clinical Professor of Medicine and Pediatrics
Chief, Section of Allergy and Immunology
Department of Medicine
by the FIRST AID authors
University of Louisville

50+ hours of high-yield online


lectures based on FIRST AID
600+ new color images & multimedia
expand key FIRST AID concepts
Exclusive FIRST AID color PDF
workbook reinforces your test prep
Watch as many times as you want
100% pass guarantee!
See website for Terms and Conditions.

www.usmlerx.com
D e i c at i o

d
n
To the contributors to this and future editions, who took time to share their
knowledge, insight, and humor for the benefit of students, residents, and
clinicians.

and

To our families, friends, and loved ones, who supported us in the task of
assembling this guide.
This page intentionally left blank
C

ontents
Authors vii

Preface ix

Acknowledgments xi

How to Contribute xiii

S EC TI O N I G e e r a l P r i c i le s 1

n
n
p

Chapter 1 Behavioral Science 3


Chapter 2 Biochemistry 17


Chapter 3 Embryology 53


Chapter 4 Microbiology 71


Chapter 5 Immunology 101


Chapter 6 Pathology 123


Chapter 7 Pharmacology 141


S EC TI O N I I r a yst e m s 159

O
g
n
S

Chapter 8 Cardiovascular 161


Chapter 9 Endocrine 197


Chapter 10 Gastrointestinal 233


Chapter 11 Hematology-Oncology 271


Chapter 12 Musculoskeletal 307


Chapter 13 Neurology 339


Chapter 14 Psychiatry 363


Chapter 15 Renal 379


Chapter 16 Reproductive 415


Chapter 17 Respiratory 451


v
S EC T I O N I I I F u ll- e th x am i at i o s 487


L
ng
E
n
n

Test Block 1 489


Test Block 2
527

Test Block 3 563



Test Block 4 599

Test Block 5 633

Test Block 6 669

Test Block 7 705

Appendix: Common Laboratory Values 741

Index 743

About the Authors 767

vi
AUTHORS

K A K L
irsten
ustad
atherine
atimer
Fellow Johns Hopkins University School of Medicine
Edmond J. Safra Center for Ethics Class of 2012
Harvard University
J L
E B

oseph
iao
Boston University School of Medicine
ike
lohm
Johns Hopkins University School of Medicine Class of 2012
Class of 2012
J L
B C , MD

erry
oo
University of Southern California Keck School of Medicine
enjamin
aplan
Resident Class of 2012
Department of Family Medicine
Boston University A M , MD

ya
ichaels
Resident
P -H C Department of Radiology
o
ao
hen
Harvard Medical School Brigham and Women’s Hospital
Class of 2012
S M , MD
L L , MD
omala
uhammed
Resident
auren
de
eon
Intern Department of General Surgery
Department of Internal Medicine
Baylor College of Medicine
The Alpert Medical School of Brown University
B zN , MD
P E
ehrou
amdari
Resident
hilip
ye
Boston University School of Medicine
Department of Psychiatry
Class of 2012
Duke University Medical Center
J G , MD
T P , MD
im
riffin
Resident
ashera
erry
Resident
Department of Surgery and Surgical Oncology
Department of Obstetrics and Gynecology
Johns Hopkins Hospital
The University of Illinois at Chicago School of Medicine
J H
C R x
ohn
egde
Harvard Medical School
hristopher
o
bury
Class of 2012 Johns Hopkins University School of Medicine
Class of 2012
E H
N S
mily
eikamp
Johns Hopkins University School of Medicine
eepa
hah
Class of 2014 Boston University
Class of 2012
T R H , MD
B S
homas
obert
ickey
Resident
ethany
trong
Department of Anesthesiology Harvard Medical School
Brigham and Women’s Hospital Class of 2012

H R. K , MD S S , MD, DMD, MPH


eenu
usarla
enry
ramer
Resident Resident
Department of Medicine Department of Oral & Maxillofacial Surgery
Massachusetts General Hospital Massachusetts General Hospital

T L J T
effrey
osoian
homas
ardaro
Johns Hopkins University School of Medicine Johns Hopkins University School of Medicine
Class of 2012 Class of 2012

vii
Jackson
V , MD M E. W

ane
arc
alker
Resident Harvard Medical School
Department of Pediatrics Class of 2012
University of California, Irvine School of Medicine

D J. V , MD
aniel
erdini
Resident
Department of Internal Medicine
University of Nevada School of Medicine at Reno

viiiviii
P

reface
With the third edition of First Aid Q&A for the USMLE Step 1, we continue
our commitment to providing students with the most useful and up-
to-date preparation guides for the USMLE Step 1. This new edition represents
an outstanding effort by a talented group of authors and includes the following:

 Almost 1000 high-yield USMLE-style questions based on the top-rated



USMLERx Qmax Step 1 Test Bank (www.usmlerx.com)
 Concise yet complete explanations to correct and incorrect answers

 Questions organized by general principles and organ systems

 Seven full-length test blocks simulate the actual exam experience

 High-yield images, diagrams, and tables complement the questions and

answers
 Organized as a perfect complement to First Aid for the USMLE Step 1

We invite you to share your thoughts and ideas to help us improve First Aid
Q&A for the USMLE Step 1. See How to Contribute, p. xiii.

Louisville Tao Le

Denver James A. Feinstein

ixix
ix
This page intentionally left blank
A
cknowledgments
This has been a collaborative project from the start. We gratefully acknowl-
edge the thoughtful comments and advice of the medical students, interna-
tional medical graduates, and faculty who have supported the authors in the
continuing development of First Aid Q&A for the USMLE Step 1.

For support and encouragement throughout the project, we are grateful to


Thao Pham, Louise Petersen, Selina Franklin, Jonathan Kirsch, and Vikas
Bhushan. Thanks to our publisher, McGraw-Hill, for the valuable assistance
of their staff. For enthusiasm, support, and commitment to this challenging
project, thanks to our editor, Catherine Johnson. For outstanding editorial
work, we thank Mary Dispenza and Emma D. Underdown. A special thanks
to Rainbow Graphics for remarkable production work.

Louisville Tao Le

Denver James A. Feinstein

xi
This page intentionally left blank
H ow
C

to
ontribute
This edition of First Aid Q&A for the USMLE Step 1 incorporates hundreds of contributions and
changes suggested by faculty and student reviewers. We invite you to participate in this process. We also
offer paid internships in medical education and publishing ranging from three months to one year (see
next page for details). Please send us your suggestions for:

 Corrections or enhancements to existing questions and explanations



 New high-yield questions

 Low-yield questions to remove

For each entry incorporated into the next edition, you will receive a $10 gift certificate, as well as per-
sonal acknowledgment in the next edition. Diagrams, tables, partial entries, updates, corrections, and
study hints are also appreciated, and significant contributions will be compensated at the discretion of
the authors.

The preferred way to submit entries, suggestions, or corrections is via our blog:
www.firstaidteam.com

Alternatively, you can email us at: firstaidteam@yahoo.com. All entries become property of the authors
and are subject to editing and reviewing. Please verify all data and spellings carefully. In the event that
similar or duplicate entries are received, only the first entry received will be used. Include a reference to
a standard textbook to facilitate verification of the fact. Please follow the style, punctuation, and format of
this edition if possible.

N N P PP N
I
TER
SHI
O
ORTU
ITIES
The First Aid Team is pleased to offer part-time and full-time paid internships in medical education and
publishing to motivated medical students and physicians. Internships may range from three months (e.g.,
a summer) up to a full year. Participants will have an opportunity to author, edit, and earn academic
credit on a wide variety of projects, including the popular First Aid and USMLERx series. Writing/
editing experience, familiarity with Microsoft Word, and Internet access are desired. For more informa-
tion, e-mail a résumé or a short description of your experience along with a cover letter to firstaidteam@
yahoo.com.

xiii
This page intentionally left blank
Section

I
General Principles

 Behavioral Science


 Biochemistry


 Embryology


 Microbiology


 Immunology

 Pathology

 Pharmacology


1
This page intentionally left blank
C
hapter 1

Behavioral Science

3
s
Hig -Yield Princi le
4 Section I: General Principles • Questions
p



Q u e st i o n s

1. Researchers investigating the development comes nervous before a test, he feels as if he




of the idiopathic inflammatory myopathies cannot move his legs. He admits that he has
h
(IIMs) such as polymyositis read that vitamin even fallen to the floor because of leg weak-
D may act as an immunomodulator that re- ness while laughing. Which of the following is
duces the development and severity of autoim- the best choice for treating this patient?
mune diseases. Given that many Americans
(A) Chloral hydrate
are vitamin D deficient, the researchers design



(B) Hydroxyzine
an observational study to assess the impact of



(C) Modafinil
vitamin D supplementation on IIM symptom



(D) Prochlorperazine maleate
severity. Subjects are surveyed at time 0 and af-



(E) Zolpidem
Behavioral Science

ter two years, and the results are listed in the



chart. Which equation represents the chance 3. A 52-year-old woman is being treated by a
of symptom improvement in subjects who took



male psychiatrist for depression stemming
vitamin D supplements relative to subjects from her recent divorce. Recently, the patient
who did not take vitamin D supplements? has been coming to her appointments dressed
up and wearing expensive perfumes. She has
also started to flirt with the doctor. The pa-
Vitamin D Vitamin D
supplement supplement tient’s demeanor and appearance had initially
taken not taken reminded the psychiatrist of his aunt. He is
Symptoms improved 50 60 uncomfortable with the patient’s new behav-
ior patterns and tells her so. She becomes very
Symptoms not
300 400 angry and storms out of the office, canceling
improved
all remaining appointments on her way out.
Reproduced, with permission, from USMLERx.com.
Which of the following behaviors is an exam-
ple of negative transference?
(A) The doctor seeing the patient as his aunt
(A) (50 / 300) / (60 / 400)


(B) The doctor telling the patient he is un-


(B) (50 / 350) / (60 / 460)


comfortable


(C) (50 / 460) / (60 / 300) (C) The patient being angry with the doctor


(D) (60 / 400) / (50 / 300)


(D) The patient dressing up for appointments


(E) (60 / 460) / (50 / 350)


(E) The patient flirting with the doctor




2. A 16-year-old boy is brought to the pediatri- 4. A 28-year-old woman presents to her primary


cian by his mother because of excessive day-


care physician because of depressed mood.
time sleepiness. She states that over the past She states that she has been depressed for as
six months she has received numerous phone long as she can remember and feels bad about
calls from the boy’s school informing her herself almost all of the time. She states that
that her son sleeps throughout all of his after- her only happy moments were during her hon-
noon classes and is often difficult to arouse at eymoon two years ago, and during a ski trip in
the end of class. The patient reports that oc- college when she felt “on top of the world.”
casionally when he wakes up in the morning She confides that for a couple weeks last
he cannot move for extended periods. He says month she felt life was no longer worth living.
that sometimes when he laughs at jokes or be- At that time, she was having extreme difficulty
Hig -Yield Princi le
Chapter 1: Behavioral Science • Questions 5

h




sleeping, a complete loss of energy, and a lack 7. The figure below is a common representation



of appetite. A review of the patient’s history used in studying the characteristics of a test’s
shows that during the past two years she has results. Using the letters in the figure, which
seen a physician for complaints of stomach up- of the following accurately describes the preva-
set, fatigue, headaches, and an unintentional lence of the disease?

p
3.6-kg (8-lb) weight gain. Physical examina-
tion and results of laboratory tests are within

s
normal limits. Which of the following is an ad- Disease
verse effect the patient may experience during + –
the course of the treatment of this illness?
(A) Agranulocytosis + W X


(B) Anorgasmia


Test

Behavioral Science
(C) Arrhythmia


(D) Polyuria


(E) Stevens-Johnson syndrome – Y Z


5. A 6-year-old girl is brought to the pediatrician


by her mother because of fecal incontinence. Reproduced, with permission, from USMLERx.com.
The mother says this behavior usually occurs
at school. According to Freud, which stage of
psychosexual development has this child failed (A) (W+X) / (W+X+Y+Z)


to progress through? (B) (W+Y) / (W+X+Y+Z)


(A) Anal stage (C) W / (W+X+Y+Z)


(D) W / (X+Y+Z)


(B) Genital stage


(E) W / (X+Z)


(C) Latency stage




(D) Oral stage
8. A 75-year-old man is recovering in the hospi-


(E) Phallic stage


tal from a left-sided below-the-knee amputa-


6. A 20-year-old man became very agitated at a tion. Three days after the surgery, the patient
suddenly develops chest pain and shortness of


party, and as a result was brought to the emer-
gency department. In the waiting room he is breath that last for 20 minutes. His pain medi-
belligerent and uncooperative. A physical ex- cation is increased, which improves the pain
amination reveals fever, tachycardia, horizon- but not the shortness of breath. X-ray of the
tal nystagmus, hyperacusis, and pupils that are chest is negative for a pulmonary embolus, so
3 mm in diameter bilaterally. Which of the the medical team decides to monitor him ex-
following substances is most likely causing the pectantly. The next day, a similar episode of
behavioral changes and physical findings ex- shortness of breath and chest pain occurs. The
hibited by this patient? patient then sustains cardiac arrest and dies.
Autopsy reveals multiple pulmonary emboli.
(A) Alcohol The family threatens to sue for malpractice for


(B) Amphetamines mismanaged postoperative care. Which of the


(C) Cocaine following is necessary to prove malpractice?



(D) Lysergic acid diethylamide
(A) A patient directly suffers harm


(E) Nicotine


(B) A physician’s presence at the time of injury


(F) Phencyclidine


(C) Intent to harm




(D) Proof beyond reasonable doubt


(E) Use of standard procedures


s
Hig -Yield Princi le
6 Section I: General Principles • Questions
p



9. A 2-month-old boy is brought to the emer- (A) 8%





gency department with respiratory insuffi- (B) 20%



ciency and failure to thrive. The pregnancy (C) 80%



and perinatal course were uneventful. Gener- (D) 85%



alized hypotonia, tongue fasciculations, and (E) 93%
h


flaccid paralysis are noted on physical exami-
nation. His hospital stay is complicated by the 11. A 66-year-old man presents to his primary care



development of tracheobronchomalacia and physician with a complaint of erectile dysfunc-
respiratory insufficiency that necessitates me- tion. His past history is significant for hyper-
chanical ventilation. Despite these efforts, tension, type 2 diabetes mellitus, peripheral
the patient dies of respiratory complications. vascular disease, and coronary artery disease
Muscle biopsy shows denervation and panfas- status postmyocardial infarction. His current
cicular atrophy. A genetics consult yields the medications are propranolol, captopril, aspirin,
Behavioral Science

pedigree shown in the image. Which of the lovastatin, metformin, fluoxetine, and sublin-
following diseases is most consistent with this gual nitroglycerin. On further questioning, he
patient’s presentation and the pedigree shown admits to wanting a prescription for sildenafil.
in the image? Which of his medications is unsafe to take
with sildenafil?

(A) Aspirin


(B) Captopril


(C) Fluoxetine


(D) Lovastatin


(E) Metformin


(F) Nitroglycerin


(G) Propranolol


12. A 17-year-old girl presents to her primary care


Reproduced, with permission, from USMLERx.com. physician with a complaint of missed menses.
A urine pregnancy test confirms that she is
pregnant. She returns to the office two weeks
(A) Becker muscular dystrophy later asking for recommendations on obtaining


(B) Duchenne muscular dystrophy an abortion. She explains that she works, lives


(C) Kugelberg-Welander disease with her husband, and is not ready for a child.


(D) Spinal muscular atrophy, type II She decides that she does not want to notify


(E) Werdnig-Hoffmann disease anyone, and says she has chosen not to talk


with her parents for many months. Her doctor
10. A new screening test for the development of understands that he must abide by her wishes


mitral regurgitation in the setting of rheumatic because she is emancipated. Which of the fol-
fever is created. A study of 1000 patients with a lowing makes this patient emancipated?
history of Streptococcus pyogenes infection and
(A) Age 17 years is considered an adult
a diagnosis of rheumatic fever is performed


(B) Full-time work
using this test, which has 90% sensitivity and


(C) High school diploma
85% specificity. The prevalence of mitral re-


(D) Living separately from her parents
gurgitation in this population is estimated to


(E) Marriage
be 40%. What is the positive predictive value


of this test?
Hig -Yield Princi le
Chapter 1: Behavioral Science • Questions 7

h




13. A group of scientists decides to conduct a study (D) Projection of the areola and papilla (with





addressing the long-term effects of maternal separate contours), and adult-type pubic
alcohol consumption on their infants after hair limited to the genital area
conception. Two hundred women, including (E) Small breast buds with elevation of breast



those who suffer from alcoholism and those papilla, and sparse, straight, downy hair on

p
who do not, are recruited into the study when the labial base
they present for their first primary care visit. A

s
medical history is taken on alcohol use, prena- 15. A 10-year-old Hispanic boy is admitted for



tal care, nutritional status, and smoking behav- bone marrow transplantation as treatment
iors; these are measured monthly during the for acute myelogenous leukemia. The doctor
pregnancy. The researchers follow the wom- wants to enroll the patient in a clinical trial
en’s pregnancies until term, after which they for a new pain medication, but both the par-
devote their attention to the health and behav- ents speak only Spanish. The consent form

Behavioral Science
iors of the offspring. Which of the following is in English, and the physician has a limited
is the most appropriate statistic the research- knowledge of Spanish. What is the physician’s
ers will be able to calculate as a result of their best option for obtaining consent from this pa-
study? tient?
(A) Attributable risk of offspring abnormalities
(A) Explain the study to the whole family in




in mothers who smoke Spanish, to the best of the physician’s abil-
(B) Odds ratio of offspring abnormalities in ity


mothers who consume alcohol during (B) Have a Spanish-speaking employee of the


pregnancy hospital translate for the patient
(C) Prevalence of alcohol consumption during (C) Have the parents sign the English form af-




pregnancy ter discussing the study via an interpreter
(D) Proportion of all offspring abnormalities (D) Obtain a translated consent form and dis-




that are due to alcohol consumption dur- cuss the study via an interpreter
ing pregnancy (E) The boy speaks English, so the parents’


(E) Relative risk of offspring abnormalities in consent will not be required


mothers who consume alcohol during
pregnancy 16. A 70-year-old man comes into his doctor’s of-


fice for a routine check-up. His past medi-
14. An 11-year-old girl is brought to the pediatri- cal history is significant for a heart attack,


cian with complaints of back pain. On physical for which he takes a daily baby aspirin and a
examination, a right thoracic scoliotic curve is b-blocker. He practices safe habits and always
noted. The pediatrician mentions to the parent wears his seat belt while driving. His health
that the development of adolescent idiopathic has been “great” for the past few years, al-
scoliosis is due to the girl being at peak height though he is concerned about his wife because
growth velocity. Peak height velocity is associ- she recently suffered a mild stroke. He denies
ated with a certain Tanner stage; what other any visual loss or motor or sensory weakness.
physical attributes would one expect to occur The patient’s physical examination is unre-
in this girl at the same time? markable. Which of the following is the lead-
ing cause of death among people age 65 years
(A) Elevation of the breast papilla only, and no
or older?


pubic hair
(B) Enlargement of the breast and areola with (A) Heart disease




a single contour, and darker, coarse, curled (B) Malignancy


pubic hair (C) Motor vehicle crashes


(C) Mature breast, and adult quantity and pat- (D) Stroke




tern of pubic hair that extends to the (E) Suicide


thighs
s
Hig -Yield Princi le
8 Section I: General Principles • Questions
p



17. A 54-year-old man with a history of poorly con- (A) The incidence rate of diabetes among





trolled hypertension complains of new-onset mothers with LGA babies is four times that
headaches. His mother passed away at an early of non-LGA mothers
age due to a stroke, and his father died of a (B) The incidence rate of LGA among women



myocardial infarction. When asked why he with diabetes is four times that of women
h
does not take better care of his blood pressure, without diabetes
he states that he is so busy with work and with (C) The incidence rate of LGA among women



the church that, by the end of the day, he often without diabetes is four times that of
forgets to take his pills. He states that he “feels women with diabetes
fine, anyway.” What ego defense mechanism is (D) The odds of diabetes among mothers with



this patient using? LGA babies is four times that of non-LGA
mothers
(A) Denial
(E) The odds of LGA among women with dia-


Behavioral Science

(B) Displacement



betes is four times that of women without


(C) Projection
diabetes


(D) Rationalization


(E) Repression
20. A 45-year-old man presents to a psychiatrist at




his wife’s prompting. He is an English profes-
18. A 3-year-old girl presents for her regular check-
sor at the University of Virginia and regularly


up. Her mother reports that she is fully toilet
wins accolades for his well-organized and ar-
trained, and that she can dress and undress
ticulate lectures. In the past three months, he
with minor assistance. She speaks in full sen-
has become convinced that his wife is having
tences, can name four colors, and can copy a
an affair with a co-worker, despite her protests
simple circle drawing. What other milestone
to the contrary. His wife recently discovered
would this child most likely have reached
that he hired a private investigator to track her
since her last visit one year ago?
whereabouts. He is very defensive when the
(A) Engages in cooperative play counselor questions his suspicions about his


(B) Has imaginary friend(s) wife. Mental status examination reveals a well-


(C) Hops on one foot dressed, middle-aged man without hallucina-


(D) Reads tions or other mood disturbances. His speech


(E) Rides a tricycle is normal and displays an appropriate affec-


(F) Stacks five blocks tive range. Which of the following is the most


likely diagnosis?
19. A retrospective cohort study is examining birth
(A) Antisocial personality disorder


complications in women with diabetes. The


study determines that babies are more likely to (B) Avoidant personality disorder


be born large for gestational age (LGA) if the (C) Delusional disorder


mother has diabetes. The relative risk for the (D) Schizoid personality disorder


study is calculated to be 4. Which of the fol- (E) Schizophrenia


lowing accurately describes this relative risk? (F) Schizophreniform disorder


Hig -Yield Princi le
Chapter 1: Behavioral Science • Answers 9

h




An s w e r s

1. The correct answer is B. This prospective, ob- 2. The correct answer is C. This patient exhib-





servational study is a cohort study. Therefore, its some of the classic symptoms of narcolepsy,

p
the likely unit of measure is the relative risk, including daytime sleepiness, cataplexy, and
which is the risk of a health outcome with a sleep paralysis. Cataplexy is defined as brief

s
given exposure versus the risk of a health out- episodes of bilateral weakness brought on by
come without the exposure. In this case, the strong emotions such as laughing or fear, with-
relative “risk” of a health outcome is really the out alteration in consciousness. Sleep paralysis
relative chance of improvement of symptoms. is an episode of partial or total paralysis that
To calculate the relative “risk,” first calculate occurs at the beginning or end of a sleep cycle.
the chance of improvement with vitamin D Patients are often aware that they are awake,

Behavioral Science
supplementation by dividing the number of but may suffer from frightening hallucina-
subjects receiving supplementation whose tions known as hypnagogic when they occur
symptoms improved (50) by the total number at sleep onset, and hypnopompic when they
of subjects taking vitamin D; this is 50/350. occur on awakening. Modafinil is an amphet-
Then calculate the chance of improvement amine derivative used to treat attention deficit/
without vitamin D supplementation by divid- hyperactivity disorder and narcolepsy. Patients
ing the number of subjects not receiving sup- suffering from cataplexy and sleep paralysis
plementation whose symptoms improved by may also benefit from the initiation of tricyclic
the total number of subjects not taking vitamin antidepressants or selective serotonin reuptake
D; this is 60 / 460. The ratio of these values is inhibitors.
the relative risk: (50 / 350) / (60 / 460).
Answer A is incorrect. Chloral hydrate is a
Answer A is incorrect. This value does not ap- nonbenzodiazepine hypnotic that is used for
propriately calculate the relative risk with vita- sedation and insomnia. This patient does not
min D supplementation. The value is equiva- need help sleeping.
lent to the odds ratio, which is the measure
Answer B is incorrect. Hydroxyzine is a non-
typically used to analyze a retrospective, case-
selective antihistamine that is used in the treat-
control study.
ment of anxiety, pruritus, nausea/vomiting, se-
Answer C is incorrect. This value does not ap- dation, and insomnia.
propriately calculate the relative risk with vita-
Answer D is incorrect. Prochlorperazine
min D supplementation. It does not represent
maleate is a typical antipsychotic used in the
any commonly used measure of analysis.
treatment of nausea, vomiting, anxiety, and
Answer D is incorrect. This value does not ap- psychosis.
propriately calculate the relative risk with vita-
Answer E is incorrect. Zolpidem is a nonben-
min D supplementation. Instead, the value is
zodiazepine hypnotic that is used in the treat-
essentially equivalent to the inverse of a calcu-
ment of insomnia.
lation for odds ratio, which is not a measure
used in data analysis. 3. The correct answer is C. Transference occurs


Answer E is incorrect. This value does not ap- when a patient projects feelings from his or her
propriately calculate the relative risk of vitamin personal life onto a doctor; countertransfer-
D supplementation. The value instead calcu- ence takes place when the doctor projects feel-
lates the chance of improvement without vita- ings onto the patient. These feelings can be
min D supplementation relative to the chance either positive or negative. The patient’s anger
of improvement with vitamin D supplementa- at the doctor when her sexual advances are re-
tion. This is the inverse of what the question buffed is an example of negative transference.
asked.
s
Hig -Yield Princi le
10 Section I: General Principles • Answers
p



Answer A is incorrect. The doctor being re- est (anhedonia), Guilt, Energy loss, Concen-
minded of his aunt by this patient is an exam- tration changes, Appetite changes, Psychomo-
ple of countertransference. tor abnormalities, and Suicidal thoughts (SIG
E CAPS). One of the symptoms has to be de-
Answer B is incorrect. The doctor telling the
pressed mood or anhedonia. Major depressive
h
patient that he is uncomfortable is not an ex-
disorder is diagnosed after a major depressive
ample of countertransference or transference.
episode without a history of mania, hypoma-
Answer D is incorrect. The patient dressing nia, or mixed episodes (when criteria for both
up for appointments is positive transference. manic and major depressive episode are simul-
Answer E is incorrect. The patient flirting taneously present for at least one week), and
with the doctor is positive transference. In its is further specified by modifiers such as recur-
most extreme form, positive transference can rent, chronic, or postpartum onset.
Behavioral Science

take the form of sexual desire. Answer C is incorrect. Arrhythmias are a


well-known adverse effect of the tricyclic anti-
4. The correct answer is D. This patient has a depressants. This class of medications, which


history of at least one major depressive episode includes imipramine, clomipramine, and
and at least one hypomanic episode without amitriptyline, work to block the reuptake of
the presence of mixed or manic episodes, a his- norepinephrine and serotonin. They are com-
tory consistent with bipolar II disorder. Treat- monly used medications in the treatment of
ment for this disorder is a mood stabilizer, major depression, obsessive compulsive disor-
most commonly lithium. A common adverse der, and fibromyalgia. However, the patient in
effect of this therapy is nephrogenic diabetes this case has a clinical history most consistent
insipidus, in which the principal cells of the with bipolar II disorder, and thus a mood sta-
renal collecting duct are unable to respond to bilizer such as lithium should be used rather
ADH secreted by the posterior pituitary. As a than an antidepressant, which could lead to
result, the patient will be unable to concen- further manic episodes in this patient.
trate urine and will thus experience frequent
urination. Answer E is incorrect. Stevens-Johnson syn-
drome is a well-known adverse effect of carba-
Answer A is incorrect. Agranulocytosis is a tox- mazepine, an anti-epileptic drug that is some-
icity associated with clozapine, an atypical an- times used to treat bipolar disorder. Although
tipsychotic. This medication is generally used this patient has a medical history consistent
in treatment of schizophrenia, but may be with bipolar II disorder, first-line treatment is
used in cases of mania that are unresponsive to generally with a different mood stabilizer such
first-line drugs such as lithium. Although the as lithium, as this medication has been proven
patient did have a history of a hypomanic epi- effective, is cheaper, and has a relatively less
sode, there is no indication for clozapine as a severe adverse-effect profile.
first-line treatment for bipolar II disorder.
Answer B is incorrect. Anorgasmia is a com- 5. The correct answer is A. The proper stage se-


mon adverse effect of selective serotonin quence of Freud’s psychosexual theory is oral,
reuptake inhibitors. This class of medication, anal, phallic, latency, and genital. Freud’s the-
which includes fluoxetine, paroxetine, sertra- ories of psychosexual development associate
line, and citalopram, is commonly used in the pleasure with certain bodily functions. Freud
treatment of major depression. Although this believed that between the ages of 18 months
patient has a history of feeling depressed, she and 3 years, children are preoccupied with
also has a history notable for a hypomanic epi- anal functions. Encopresis is fecal inconti-
sode. A major depressive episode is diagnosed nence and can range from mild to severe.
if the patient has 5/9 symptoms for at least two Answer B is incorrect. Freud’s genital stage
weeks, including Sleep changes, loss of Inter- encompasses adolescents to adults and is char-
Hig -Yield Princi le
Chapter 1: Behavioral Science • Answers 11

h




acterized by the desire to achieve sexual grati- Answer C is incorrect. Patients presenting
fication. with acute cocaine intoxication will show
symptoms of euphoria, psychomotor agitation,
Answer C is incorrect. Freud’s latency stage
impaired judgment, tachycardia, pupillary di-
encompasses the ages of 6-12 years and is char-
lation, hypertension, hallucinations, paranoid
acterized by a suppression of sexual desire.

p
ideations, angina, and sudden cardiac death.
Answer D is incorrect. Freud’s oral stage en- On withdrawal, they will show a post-use

s
compasses birth to the age of 18 months. “crash” that includes severe depression, hyper-
Freud believed that children of this age gain somnolence, fatigue, malaise, and severe psy-
satisfaction from oral functions. chological craving.
Answer E is incorrect. Freud’s phallic stage Answer D is incorrect. Patients presenting
encompasses the ages of 3-5 years and is most with acute lysergic acid diethylamide intoxica-
commonly known as the oedipal phase. Freud

Behavioral Science
tion will display marked anxiety or depression,
believed that at this stage children begin to de- delusions, visual hallucinations, flashbacks,
velop sexual fantasies. and pupillary dilation.

6. The correct answer is F. This patient has Answer E is incorrect. Patients presenting
with acute nicotine intoxication will show


taken phencyclidine, or PCP. Patients with
PCP intoxication show signs of belligerence, symptoms of restlessness, insomnia, anxiety,
impulsiveness, fever, psychomotor agitation, and arrhythmias. On withdrawal, they will
vertical and horizontal nystagmus, tachycardia, have symptoms of irritability, headache, anxi-
ataxia, homicidality, psychosis, and delirium. ety, weight gain, craving, and tachycardia.
On withdrawal, patients may demonstrate a
7. The correct answer is B. The prevalence is
recurrence of intoxication when the PCP,


the number of individuals with a disease in a
which was trapped in an ionized form in the
given population at a given time. Prevalence is
acidic gastric lumen, is reabsorbed in the alka-
estimated by test results but is not a measure of
line duodenum. PCP users will have normal
a test’s validity. In the chart shown, the preva-
or small pupils. Death can result from a variety
lence can also be determined by calculating
of causes, including respiratory depression and
the number of true-positive plus false-negative
violent behavior.
results divided by the total number of patients.
Answer A is incorrect. Patients presenting with
Answer A is incorrect. This term represents
acute alcohol intoxication will show symptoms
the incidence of positive test results.
of disinhibition, emotional lability, slurred
speech, ataxia, coma, and blackouts. On with- Answer C is incorrect. This represents true-
drawal, they will demonstrate a tremor, tachy- positive results divided by the total number of
cardia, hypertension, malaise, nausea, seizures, patients. This would be the percent of true-
delirium tremens, tremulousness, agitation, positive results of all tested, but it is not used
and hallucinations. very often.
Answer B is incorrect. Patients presenting Answer D is incorrect. This represents true-
with amphetamine intoxication will display positive results divided by the total number of
psychomotor agitation, impaired judgment, patients tested less those with true-positive re-
pupillary dilation, hypertension, tachycardia, sults, and would not be a meaningful calcula-
euphoria, prolonged wakefulness and atten- tion.
tion, cardiac arrhythmias, delusions, hallucina-
Answer E is incorrect. This represents the
tions, and fever. On withdrawal, they will show
number of true-positive results over the to-
a post-use “crash” that includes depression,
tal number of patients without disease. This
lethargy, headache, stomach cramps, hunger,
would not be a meaningful calculation.
and hypersomnolence.
s
Hig -Yield Princi le
12 Section I: General Principles • Answers
p



8. The correct answer is A. Malpractice suits fascicles (panfascicular atrophy). Unlike SMA


require that the patient prove dereliction, types II and III, this patient’s disease developed
damage, and direct harm by a physician with at an early age, so early milestones were not
whom there was an established relationship. achieved. This is not the case in the less severe
Direct harm is a concept that the injury is forms of SMA.
h
causally related to the actions of the physician.
Answer A is incorrect. BMD involves the
This is also known as proximal cause, and in
same genetic locus that is affected in DMD,
many cases is the most difficult aspect to prove,
but its occurrence is less common. It follows a
as a temporal relationship does not necessarily
more indolent course, with onset often occur-
imply a causal relationship.
ring in late childhood.
Answer B is incorrect. It is not necessary for
Answer B is incorrect. Duchenne muscular
the doctor to have been present at the time of
dystrophy (DMD) and Becker muscular dystro-
Behavioral Science

injury, but there must be an established rela-


phy (BMD) are both characterized by defects
tionship between the physician and patient.
in the 427-kDa protein dystrophin, encoded on
Answer C is incorrect. Intent is not a factor the Xp21 region. DMD is the most common
in malpractice proceedings. These proceed- form of muscular dystrophy, with an incidence
ings are civil lawsuits, not criminal. As such, of about one in 3500 live births. Onset typi-
when intent or gross misconduct is proven, cally occurs after infancy and before the age
additional punitive damages may be assessed of five. The clinical course is characterized by
against the physician. progressive muscle weakness and wasting that
lead to wheelchair dependence by 10-12 years
Answer D is incorrect. As malpractice suits
of age. Early motor milestones are met in pa-
are civil rather than criminal proceedings, the
tients with BMD and DMD.
plaintiff is required only to prove “more likely
than not” that the actions of the defendant led Answer C is incorrect. Type III spinal muscu-
to damages. lar atrophy, or Kugelberg-Welander disease, is
characterized by the onset of proximal muscle
Answer E is incorrect. Proof of malpractice re-
weakness after the age of 2 years, the ability
quires dereliction, or deviation from standard
to walk independently until the disease pro-
procedure, that leads to the injury in question.
gresses, and survival into adulthood.
9. The correct answer is E. Spinal muscular Answer D is incorrect. Type II spinal mus-


atrophy (SMA) is one of the most common cular atrophy is characterized by the onset of
autosomal-recessive diseases, affecting ap- proximal muscle weakness before 18 months
proximately one in 10,000 live births. It has a of age, the ability to sit but not to walk un-
carrier frequency of approximately one in 50 aided, and survival beyond 4 years of age.
and is characterized by symmetric proximal
muscle weakness due to the degeneration of 10. The correct answer is C. Positive predictive


the anterior horn cells of the spinal cord. SMA value (PPV) is the probability that a positive
is classically divided into three subtypes based test result is truly positive. It can be calculated
on age of onset and clinical severity. Type I by taking the number of true-positive results
SMA (Werdnig-Hoffmann disease), the most and dividing it by the total positive results.
severe, is characterized by the onset of signifi- Since the prevalence is 40%, the number of
cant muscle weakness and hypotonia in the positive patients will be 0.4 × 1000 = 400 and
first few months of life, and the inability to the number of negative patients will be 1000 -
sit or walk. Manifestations may even occur in 400 = 600. The number of true-positives can
utero with reduced fetal movement. Fatal res be found by multiplying the sensitivity by this
­
piratory failure usually occurs before the age total, giving us 0.9 × 400 = 360. The number
of 2 years. Muscle biopsy demonstrates large of false-positives can be found by multiplying
numbers of atrophic fibers that involve entire the specificity by the total negative patients
Hig -Yield Princi le
Chapter 1: Behavioral Science • Answers 13

h




and subtracting that product from the total tension-related adverse effects. However, the
negatives, or 600 - (0.85 × 600) = 90. Then, captopril is probably contributing to his erec-
360 / (360 + 90) = 0.8, or 80%. Remember: tile dysfunction.
PPV and negative predictive value change
Answer C is incorrect. Fluoxetine and silden
with prevalence in a population, so the esti-

­
afil have no known dangerous interactions.

p
mated prevalence must be taken into account
However, the fluoxetine is probably contribut-
when calculating the number of true-positives

s
ing to his erectile dysfunction.
and true-negatives.
Answer D is incorrect. Lovastatin and silden
Answer A is incorrect. Dividing the number of

­
afil have no known dangerous interactions.
false-negative results by the total negative re-
sults would give an answer of 8%. Answer E is incorrect. Metformin and silden

­
afil have no known dangerous interactions.
Answer B is incorrect. Dividing the number

Behavioral Science
of false-positive (rather than true-positive) find- Answer G is incorrect. Propranolol and
ings by the total positive results would give an sildenafil have no known dangerous interac-

­
answer of 20%. tions, though the combination may increase
the risk of hypotension related adverse effects.
Answer D is incorrect. Switching the values
However, the propranolol may be contributing
for sensitivity and specificity would give an an-
to his erectile dysfunction.
swer of 85%.
Answer E is incorrect. The negative predictive 12. The correct answer is E. Emancipation is a


value is 93%. legal definition through which minors become
independent of their parents and are free to
11. The correct answer is F. As is common for make medical decisions for themselves. A mi-


many older patients, this man is taking several nor, which is a legal condition defined by age,
prescription medications. Though a couple of can generally acquire emancipation through
his prescriptions should be used with caution court order or marriage. These situations usu-
with sildenafil (captopril and propranolol), the ally suggest that the minor will be financially
only one that might significantly interact with independent of his or her parents. This patient
sildenafil is sublingual nitroglycerin. Nitroglyc- is married and is therefore emancipated.
erin is used for prompt relief of an ongoing at-
Answer A is incorrect. While this patient has
tack of angina precipitated by exercise or emo-
many adult responsibilities, 18 years is the le-
tional stress. Nitrates relax vascular smooth
gal age of consent and adulthood.
muscle by their intracellular conversion to
nitrite ions and then to nitric oxide, which in Answer B is incorrect. Full-time work suggests
turn activates cGMP and increases the cell’s that the patient is financially independent, but
cGMP level. Elevated cGMP ultimately taken alone it is not proof of emancipation.
causes vascular smooth muscle relaxation. Answer C is incorrect. A high school diploma
This is the same mechanism sildenafil uses to does not provide emancipation. Even though a
cause smooth muscle relaxation and increased minor becomes the primary decision maker af-
blood flow into the corpus cavernosum at a ter high school graduation, he or she is not nec-
certain level of sexual stimulation. Using these essarily financially independent of the parents.
drugs together can lead to severe hypotension
and cardiovascular collapse. Answer D is incorrect. A teenager may state
he or she has separated from the parents, but
Answer A is incorrect. Aspirin and sildenafil unless the courts have approved a legal separa-
have no known dangerous interactions. tion, merely saying she is “separated” from her
Answer B is incorrect. Captopril and sildenafil parents is not enough; legally the parents are
have no known dangerous interactions, though still financially responsible for the child until
the combination may increase the risk of hypo- he or she turns 18.
s
Hig -Yield Princi le
14 Section I: General Principles • Answers
p



13. The correct answer is E. The study described the prevalence of the risk factor in the popula-


here is a cohort study, because it includes a tion, which would not be known in this study.
group with and a group without a given risk
factor (fetal exposure to alcohol) and then 14. The correct answer is B. Tanner stage 3 is the



looks at whether the risk factor changes the stage when most girls experience peak height
h
chances of offspring getting the disease (abnor- velocity. Peak height velocity occurs approxi-
malities). The study is prospective, because the mately one year after the initiation of breast de-
group members are looked at before the dis- velopment. Also, pubic hair becomes dark and
ease (abnormality) develops in the offspring. curly during this stage.
Relative risk can be calculated from the results Answer A is incorrect. This description corre-
of a cohort study by comparing the rate of dis- sponds to Tanner stage 1.
ease in the group with the risk factor to the rate
of disease in the group without the risk factor. Answer C is incorrect. This description corre-
Behavioral Science

sponds to Tanner stage 5.


Answer A is incorrect. Attributable risk can be
calculated from the results of a cohort study Answer D is incorrect. This description corre-
and describes the proportion of disease that is sponds to Tanner stage 4.
due to the risk factor under study. Although Answer E is incorrect. This description corre-
smoking behavior of the women is being re- sponds to Tanner stage 2.
corded, the study is not designed to look at
the impact of this risk factor on fetal abnor- 15. The correct answer is D. Obtaining informed
mality; the rate of smoking in the two groups


consent from the patient means that the pa-
of women is unknown, and thus we do not tient understands the risks, benefits, and al-
know whether there are sufficient numbers ternatives to the study, and that the doctor re-
of women in the “exposed” and “unexposed” lays to the patient pertinent matters about the
groups when it comes to tobacco. plan of care. For the non-English-speaking
Answer B is incorrect. An odds ratio is similar patient, the consent is translated into the ap-
to relative risk, but it is calculated from the re- propriate language and discussed with him/her
sults of a case-control study, not from a cohort through an interpreter. This allows the patient
study. Because birth abnormality is a relatively (or in this case, his parents) freedom to read
rare outcome, the odds ratio from a case-control and process the consent and to discuss it later.
study would likely closely approximate the ac- Whereas this option may not be possible for
tual relative risk. every language or reasonable for every study, it
is appropriate in this non-emergent situation.
Answer C is incorrect. Prevalence is a mea-
sure of how many cases of a given disease exist Answer A is incorrect. With limited knowl-
in a population that is at risk for that disease. edge of Spanish, the doctor will unlikely be
This is not the best answer in this case, be- able to address all the important issues delin-
cause there are no data to judge whether this eated in the consent form.
group of 200 women represents the true preva- Answer B is incorrect. Having someone other
lence of alcoholism during pregnancy in the than an interpreter translate will be invading
community. patient privacy, incomplete, and not perfectly
Answer D is incorrect. The statistic described accurate/reliable.
in this answer is the population attributable Answer C is incorrect. In a non-emergent set-
risk (PAR), which helps us understand, in a ting, the best approach is to allow the patient/
given population, how much less common a family to view a translated copy of the consent
disease (fetal abnormality) would be if a given and consider all their options in an unbiased
risk factor (alcohol consumption during preg- manner. The use of an interpreter, however,
nancy) were completely eliminated. To calcu- would be invaluable in an emergent setting.
late PAR, the attributable risk is multiplied by
Hig -Yield Princi le
Chapter 1: Behavioral Science • Answers 15

h




Answer E is incorrect. The patient is too ently more or less logical explanation for an
young to give consent (<18 years). act or decision actually produced by uncon-
scious impulses. This patient’s assertion that he
16. The correct answer is A. Heart disease is the “feels fine, anyway” may be a form of rational-


leading cause of death among the elderly (65 ization, but the primary ego defense mecha-

p
years old and older), as well as the leading nism that he is using is denial.
cause of death if all ages are combined. The

s
Answer E is incorrect. Repression is the un-
patient is at a particularly high risk for subse-
conscious exclusion of a painful or anxiety-
quent cardiac events due to his previous history
provoking thought, impulse, or memory from
of myocardial infarction. Other major risk fac-
awareness.
tors for cardiac events are high blood pressure,
hypercholesterolemia, and diabetes mellitus.
18. The correct answer is E. This is a typical



Answer B is incorrect. Cancer is the second well-child visit. Other milestones reached at

Behavioral Science
most common cause of death. It ranks after approximately this age include stacking nine
heart disease, but before stroke. blocks, riding a tricycle, and beginning to en-
gage in group play. Riding a tricycle at 3 years
Answer C is incorrect. Motor vehicle crashes,
is easy to remember because a tricycle has
and accidents in general, are leading causes of
three wheels. The number of blocks stacked
death. However, they are the most common
between ages 2 and 4 years is about three times
cause of death among children (1-14 years
the child’s age in years; that is, a 2-year-old can
old) and adolescents (15-24 years old) and not
stack six blocks, whereas a 3-year-old can stack
among the elderly.
nine blocks.
Answer D is incorrect. Stroke is the third most
Answer A is incorrect. Engaging in coopera-
common cause of death among the elderly, be-
tive play is achieved between 4 and 5 years of
hind heart disease and then cancer.
age.
Answer E is incorrect. Suicide is not the lead-
Answer B is incorrect. Imaginary friends are
ing cause of death in the elderly. It is a com-
typically present between 4 and 5 years of age.
mon cause of death among adolescents.
Answer C is incorrect. Hopping on one foot is
17. The correct answer is A. The mechanism of typical of a 4-year-old child.


denial is when one fails to recognize the obvi-
Answer D is incorrect. Reading is most often
ous implications or consequences of a thought,
learned at 5-6 years of age and older.
act, or situation. This ego defense mechanism
is often seen in patients with recently diag- Answer F is incorrect. Children at 2 years of
nosed HIV or cancer. age can usually stack six blocks, whereas chil-
dren at 3 years can stack nine blocks.
Answer B is incorrect. Displacement is a de-
fense mechanism whereby ideas and feelings
19. The correct answer is B. A retrospective co-
that a patient wishes to avoid are transferred


hort study includes a group of subjects who
to another person or object; for example, a pa-
had a certain condition or received a certain
tient who yells at the nurse because he is angry
treatment at some time in the past and com-
at news he has just received from the doctor.
pares their outcomes to those of another group
Answer C is incorrect. Projection is the pro- (a control group) made up of subjects who did
cess of attributing one’s thoughts or impulses, not have this condition or receive the treat-
usually ones that are unacceptable or undesir- ment. In this study the risk factor is the pres-
able, to another person. ence of diabetes in the mothers and the out-
come is LGA babies. The incidence of LGA
Answer D is incorrect. Rationalization pro-
births in women with diabetes is four times
duces a more socially acceptable and appar-
that in women without diabetes. Relative risk is
s
Hig -Yield Princi le
16 Section I: General Principles • Answers
p



defined as the incidence rate of some outcome Answer A is incorrect. Cluster B personality
in those exposed to a risk factor divided by the disorders include antisocial, borderline, histri-
incidence rate of those not exposed. This defi- onic, and narcissistic types. Patients with anti-
nition gives the factor at which the incidence social personality disorder show a disregard for
rate of LGA among women with diabetes is and often violate the rights of others. These in-
h
larger than the incidence rate of LGA among dividuals often have a criminal history. This is
women without diabetes. the only personality disorder with an age limit
(18 years). Minors with similar behavior are
Answer A is incorrect. This choice describes
classified as having conduct disorder.
the correct type of risk analysis but describes
the relationship in reverse. Answer B is incorrect. Cluster C personal-
ity disorders are characterized by anxiety and
Answer C is incorrect. This choice reverses
include avoidant, obsessive-compulsive, and
the findings of the study, which shows that
Behavioral Science

dependent personality types. Individuals with


the incidence of LGA is four times more in
avoidant personality disorder are sensitive to
women with diabetes.
rejection, are socially inhibited, and have over-
Answer D is incorrect. This choice incorrectly whelming feelings of inadequacy.
uses odds rather than incidence rates and also
Answer D is incorrect. Patients with schiz-
describes the relationship of the findings of the
oid personality disorder exhibit voluntary so-
study in reverse.
cial withdrawal (unlike avoidant patients) and
Answer E is incorrect. This choice describes have limited emotional expressions. They lack
an odds ratio for a case-control study. A case- strange beliefs and thoughts of the schizotypal
control study evaluates the presence of risk personality disorder.
factors in people with and without a disease.
Answer E is incorrect. Schizophrenia is a
Although this is the opposite of a cohort study,
chronic psychiatric condition. It affects 1% of
the results are still reported in terms of disease
the population and usually begins before age
presence with respect to risk factors; that is, the
25. The Diagnostic and Statistical Manual
presence or absence of disease is categorized
of Mental Disorders, Fourth Edition, specifies
in the group with risk factors and compared to
the active phase of the disease and requires
the group without risk factors. The difference,
that at least two of the following symptoms be
however, is that odds are used rather than inci-
present during a one-month period: delusions,
dence. The incidence rate is a percentage (eg,
hallucinations, disorganized speech, grossly
50 out of 100). Odds are calculated by dividing
disorganized or catatonic behavior, and nega-
those with disease by those without (50 to 50,
tive symptoms (eg, flat affect, lack of motiva-
or 1 to 1).
tion, or poverty of speech). Moreover, signs of
20. The correct answer is C. Delusional disor- the disturbance must be present for at least six
months, such as one of the above symptoms in


der is diagnosed following one month of non-
an attenuated form (eg, magical thinking, so-
­
bizarre delusions that are usually focused
around a particular topic, in this case, the fi- cial withdrawal, or other negative symptoms).
delity of the patient’s wife. The delusions are Answer F is incorrect. Schizophreniform dis-
not attributable to another psychiatric disorder order is similar to schizophrenia except that
such as schizophrenia. Delusional disorder its symptoms have lasted between one and six
does not markedly impair the person’s func- months. In contrast, patients with schizophre-
tioning in daily activities. The ramifications nia must have had symptoms for longer than
are limited to the delusional content. six months.
Chapter 2

Biochemistry

17
Hig -Yield Princi les
18 Section I: General Principles • Questions
p



Q u e st i o n s

1. A 6-year-old boy presents to his pediatrician 3. A 54-year-old woman complains of fatigue,





with skin lesions all over his body. For several difficulty climbing stairs, and weight loss. Her
h
years he has been very sensitive to sunlight. medical history is notable for hypertension.
Neither the boy’s parents nor his siblings have She takes no medications. Her skin is moist
the same skin lesions or sun sensitivity. Biop- and she has a prominent stare. The patient’s vi-
sies of several of the boy’s lesions reveal squa- tal signs are:
mous cell carcinoma. Which mutation would
Heart rate: 112/min
one expect to see in this patient’s DNA?
Blood pressure: 143/90 mm/Hg
(A) Methylation of the gene Respiratory rate: 14/min


(B) Missense mutation in the gene Oxygen saturation: 98% on room air


(C) Nonsense mutation in the middle of the
Biochemistry

Laboratory tests reveal markedly elevated levels




gene
of a specific hormone. What is the first mole-
(D) Point mutation within the enhancer region
cule produced within a cell in response to this


(E) Point mutation within the operator region
hormone?


(F) Point mutation within the promoter region


(G) Thymidine dimers
(A) Carbohydrate




(B) DNA


2. A metabolic process is pictured below. Which (C) Fatty acid




intermediate in this process inhibits the rate- (D) Protein


limiting enzyme of glycolysis and activates the (E) RNA


rate-limiting enzyme of fatty acid synthesis?
4. A 35-year-old man presents to the physician


Pyruvate
with arthritic pain in both knees along with
back pain. He states that the pain has been
Acetyl-CoA present for months. In an effort to obtain re-
lief, he has taken only aspirin, but this has
D B been of little benefit. The patient is afebrile,
NADH
cis-aconitate
and his slightly swollen knee joints are neither
hot nor tender to palpation; however, the pain
C Isocitrate
does restrict his motion. The cartilage of his
CO2 + NADH
Fumarate ears appears slightly darker than normal. No
FADH2
A tophi are present. A urine specimen is taken
CO2 + NADH for analysis of uric acid content and turns
Succinate
GTP black in the laboratory while standing. A de-
+
CoA E fect in which of the following is the most likely
underlying cause of the patient’s condition?
Reproduced, with permission, from USMLERx.com. (A) a-Ketoacid dehydrogenase


(B) Galactokinase


(C) Homogentisic acid oxidase
(A) A


(D) Orotate phosphoribosyl transferase


(B) B


(E) Phenylalanine hydroxylase


(C) C




(D) D


(E) E


Hig -Yield Princi les
Chapter 2: Biochemistry • Questions 19

h




5. A patient who is a carrier of sickle cell trait 6. A 57-year-old woman visits her primary care





presents to the clinic. The single base-pair mu- physician. Laboratory studies reveal an LDL of
tation for sickle cell anemia destroys the MstII 194 mg/dL and HDL of 41 mg/dL. Her phy-
restriction enzyme recognition site represented sician begins therapy with a drug that inhibits
by an asterisk in the image. The restriction production of mevalonic acid. Which of the

p
enzyme-binding sites are shown as arrows on following is a common side effect of this ther-
the map. DNA from this patient is treated with apy?
MstII and run on an electrophoresis gel. The
(A) Hepatomegaly without elevations in aspar-
DNA is then hybridized with a labeled probe



tate aminotransferase or alanine amino-
that binds to the normal gene in the posi-
transferase
tion shown on the map. In the Southern blot
(B) Muscle injury clinically similar to myositis
shown in the image, which lane represents the



(C) Spontaneous abortion of a pregnancy
patient?



(D) Suicidality and homicidality in patients



with bipolar disorder

Biochemistry
βA Globin gene (E) Tonic-clonic seizures



MstII Probe MstII MstII
A B C D E F 7. A nucleic acid fragment is added to four dif-


1.15 Kb 0.2 Kb 1.35 Kb ferent tubes along with a polymerase, a radio-
MstII MstII 1.15 Kb labeled primer, and deoxynucleotides. Each
0.2 Kb
* tube also contains one of the four bases as
βS Globin gene dideoxynucleotides. The four tubes are then
run on electrophoresis gel and visualized by
Reproduced, with permission, from USMLERx.com.
autoradiography. For which of the following
purposes would the described laboratory tech-
(A) A nique be utilized?


(B) B (A) To amplify DNA fragments




(C) C (B) To create an allele-specific oligonucleotide




(D) D probe


(E) E (C) To decipher the order of nitrogenous bases




(F) F in the human genome


(D) To determine the base pairing of a seg-


ment of DNA with a DNA probe
(E) To determine the base pairing of a seg-


ment of RNA with a DNA probe
(F) To establish the presence of a given pro-


tein
(G) To show the presence of a specific anti-


body in plasma
Hig -Yield Princi les
20 Section I: General Principles • Questions
p



8. A 45-year-old white woman presents to her regulated both across the cell membrane and


physician complaining of several months of within the cell via sequestration in the en-
worsening shortness of breath. Previously she doplasmic reticulum and mitochondria. In
was told she had asthma because she was hav- which of the following ways does increased
ing intermittent episodes of wheezing com- intracellular calcium concentration cause the
h
bined with a productive cough and difficulty most cell damage?
catching her breath. She used to run two miles
(A) Enzyme activation
every morning but can no longer walk more



(B) Free radical generation
than 10 city blocks without stopping. She has



(C) Increased membrane permeability
never smoked. On physical examination she is



(D) Inhibition of glycolysis
using her accessory muscles to assist with res-



(E) Inhibition of oxidative phosphorylation
piration. Pulmonary examination is notable



for an increased decreased FEV1/FVC ratio, 10. A scientist working in a research laboratory has
decreased air movement with each breath, and



been examining different agonists of serotonin
Biochemistry

increased resonance upon percussion. X-ray of receptor 1B (5-HT1B), a G-protein-coupled re-


the chest is shown in the image. Which of the ceptor. Compound A has a much higher affin-
following is the most likely underlying cause ity for 5-HT1B than compound B. Both com-
for this patient’s disease? pounds have a higher affinity for the receptor
than serotonin. Which of the following de-
scribes the relationship between compound A
and compound B when considering the guan

­
ine-nucleotide exchange activity of 5-HT1B?
(A) Km for the exchange reaction with com-


pound A is higher than that with com-
pound B
(B) Km for the exchange reaction with com-


pound A is lower than that with compound
B
(C) Km values with compounds A and B are


the same
(D) The maximum reaction rate with com-


pound A is greater than that with com-
Courtesy of Dr. James Heilman. pound B
(E) The maximum reaction rate with com-


pound B is greater than that with com-
(A) A genetic mutation resulting in deficient pound A


levels of a protease
(B) A genetic mutation resulting in deficient 11. A mother brings her 6-month-old son to the




levels of a protease inhibitor pediatrician. She has noticed that he seems
(C) A mutation in the p53 gene “afraid of light” and, after some Internet re-


(D) A mutation of the CFTR gene, which en- search, she is concerned that he might be an


codes a regulated chloride channel albino. Laboratory analysis reveals uropor-
(E) Airway inflammation, airflow obstruction, phyrin in his urine. The child most likely has


and bronchial hyperresponsiveness which of the following conditions?
(A) Deficiency of coproporphyrinogen oxidase
9. As increased intracellular calcium is detrimen-


(B) Deficiency of porphobilinogen deaminase


tal to the cell, calcium homeostasis is tightly


(C) Deficiency of uroporphyrinogen decarbox-


ylase
Hig -Yield Princi les
Chapter 2: Biochemistry • Questions 21

h




(D) Inhibition of ferrochelatase and (A) 3-Hydroxy-3-methylglutaryl coenzyme A





δ-aminolevulinic acid dehydrase reductase
(E) Overexpression of porphobilinogen deami- (B) Adenosine deaminase





nase (C) Aldose reductase



(D) Galactose-1-phosphate uridyltransferase

p


12. A 48-year-old woman of Mediterranean de- (E) Hexokinase




scent presents because of fatigue, arthralgias, (F) Insulin-like growth factor



discomfort in her right upper abdominal quad-
rant, and polyuria. Laboratory tests are remark- 14. Acquired mutation in the p53 gene is the most



able for elevated glucose level, elevated biliru- common genetic alteration found in human
bin, low hemoglobin, elevated reticulocytes, cancer (> 50% of all cancers). A germline
and increased transferrin saturation. Cardiac mutation in p53 is the causative lesion of Li-
testing shows moderate restrictive cardiomyop- Fraumeni familial cancer syndrome. In many
athy. She frequently has required blood trans- tumors, one p53 allele on chromosome 17p is

Biochemistry
fusions throughout her life. Which hereditary deleted and the other is mutated. What type of
disorder does this patient most likely have? protein is encoded by the p53 gene?
(A) Absence of the hemoglobin a-chain (A) Caspase




(B) Absence of the hemoglobin b-chain
(B) DNA repair enzyme




(C) Mutation resulting in increased absorption (C) Membrane cell adhesion molecule




of dietary iron (D) Serine phosphatase


(D) Mutations in the gene encoding ankyrin (E) Telomerase




(E) Mutations resulting in copper accumula- (F) Transcription factor




tion (G) Tyrosine kinase


13. A 52-year-old man with a 12-year history of 15. A segment of DNA is isolated and added to a




poorly controlled diabetes mellitus presents to mixture of four deoxynucleotides, two specific
his physician complaining of changes in his vi- oligonucleotide sequences, and heat-stable
sion. Physical examination reveals opacities on DNA polymerase. The mixture is then heated
the lens of the eye similar to those seen in this to denature the DNA, cooled, and reheated
image. Which enzyme most likely contributed in a number of cycles. Which of the following
to this complication? laboratory techniques does this describe?
(A) Enzyme-linked immunosorbent assay


(B) Gel electrophoresis


(C) Northern blot


(D) Polymerase chain reaction


(E) Sequencing


(F) Southern blot


(G) Western blot


Courtesy of Dr. Rakesh Ahuja.
Hig -Yield Princi les
22 Section I: General Principles • Questions
p



16. A 32-year-old woman presents to her physician (C) Microorganism’s ability to produce b-lacta-





for the third time in six months. She has been mase
feeling very tired and depressed, and has come (D) Mutation in terminal amino acid of micro-



to talk about starting antidepressants. She also organism’s cell wall component
complains of a 4.5-kg (10-lb) weight gain over (E) Mutation in the microorganism’s penicil-
h


the past three months. During her physical lin-binding protein
examination the physician notices that she is
wearing a sweater and a coat, despite the room 18. A 2-year-old boy presents to the pediatrician



being at a warm temperature. Problems with with fever, facial tenderness, and a green, foul-
the thyroid are suspected, and a biopsy is per- smelling nasal discharge. The patient is diag-
formed (see image). This woman may have a nosed with sinusitis, and the physician notes
human leukocyte antigen subtype that also in- that he has a history of recurrent episodes of
creases her risk of which disease? sinusitis. X-ray of the chest is ordered because
of the fever; it reveals some dilated bronchi
Biochemistry

and shows the heart situated on the right side


of his body. A congenital disorder is diagnosed.
Which other finding would this patient be
most likely to have?

(A) Defective chloride transport


(B) Elevated blood sugar


(C) Infertility


(D) Reactive airway disease


(E) Tetralogy of Fallot


19. A 5-day-old boy is brought to the emergency


department after a tonic-clonic seizure at
home. The infant is the product of a full-term,
uneventful pregnancy, and was normal until
Reproduced, with permission, from USMLERx.com. two days prior to presentation. The mother re-
ports irritability and poor feeding at home, and
the infant was difficult to rouse this morning
(A) Multiple sclerosis
before suffering the seizure. On physical ex-


(B) Pernicious anemia
amination, the infant is tachypneic to 75/min,


(C) Psoriasis
has icteric sclerae, and has poor muscle tone


(D) Steroid-responsive nephrotic syndrome
throughout. Laboratory studies show the fol-


lowing levels: plasma ammonia, 300 µmol/L
17. A 65-year-old woman who has been in the hos-
(normal = 10-40 µmol/L); blood urea nitro-


pital for three weeks receiving cefotaxime to
gen, 1.5 mg/dL; and creatinine, 0.4 mg/dL.
treat Klebsiella pneumonia develops a urinary
A plasma amino acid analysis fails to detect
tract infection. Urine cultures are positive for
citrulline. Urine amino acids demonstrate el-
Enterococcus faecium. Treatment with vanco-
evated orotic acid levels. This patient suffers
mycin is attempted but is unsuccessful. Which
from a deficiency of which of the following en-
of the following aided in this microorganism’s
zymes?
ability to persist despite vancomycin treat-
ment? (A) a-Galactosidase A


(B) Aldose B
(A) Alteration of microorganism’s gyrase


(C) Galactose 1-phosphate uridylyltransferase


(B) Methylation of microorganism’s rRNA at a


(D) Lysosomal a-glucosidase


ribosome-binding site


(E) Ornithine transcarbamylase


Hig -Yield Princi les
Chapter 2: Biochemistry • Questions 23

h




20. A 42-year-old woman presents to her physician gin. X-ray of the chest reveals cardiomegaly.


with generalized itching. Physical examination He has a difficult time sitting upright and
reveals scleral icterus. Laboratory tests show: cannot squeeze the physician’s fingers or the
ring of his pacifier with any noticeable force.
Total bilirubin: 2.7 mg/dL
Despite a number of interventions, the child’s
Conjugated bilirubin: 2.4 mg/dL

p
symptoms continue to worsen until his death
Alkaline phosphatase: 253 U/L
two weeks later. On autopsy, it is likely that this
Aspartate aminotransferase: 36 U/L
patient’s cells will contain an accumulation of
Alanine aminotransferase: 40 U/L
which of the following substances?
What is the most likely mechanism underlying
(A) Glucose
this patient’s jaundice?



(B) Glycogen



(A) Absence of UDP-glucuronyl transferase (C) Oxaloacetate




(B) Decreased levels of UDP-glucuronyl trans- (D) Pyruvate




ferase (E) Urea

Biochemistry


(C) Extravascular destruction of the patient’s


RBCs 23. After consumption of a carbohydrate-rich



(D) Intrahepatic or extrahepatic biliary ob- meal, the liver continues to convert glucose to


struction glucose-6-phosphate. The liver’s ability to con-
(E) Intravascular destruction of the patient’s tinue this processing of high levels of glucose


RBCs is important in minimizing increases in blood
glucose after eating. What is the best explana-
21. A 5-year-old boy was playing outside during tion for the liver’s ability to continue this con-


recess when he began to experience difficulty version after eating a carbohydrate-rich meal?
breathing. He was brought to his physician,
(A) The hepatocyte cell membrane’s permea-
because his symptoms seemed to be getting


bility for glucose-6-phosphate
worse. On examination, the physician notes
(B) The high maximum reaction rate of gluco-
that the boy is struggling to breathe and hears


kinase
diffuse wheezing bilaterally. The boy’s heart
(C) The high maximum reaction rate of hexo-
rate is 98/min, respiratory rate is 24/min, and


kinase
oxygen saturation is 90%. His medical history
(D) The high Michaelis-Menten constant of
is significant only for seasonal allergies and


hexokinase
mild eczema. Which type of medication will
(E) The low Michaelis-Menten constant of
alleviate this patient’s respiratory symptoms?


glucokinase
(A) b1 Antagonist


(B) b1 Agonist 24. A 30-year-old man is diagnosed with type I fa-




(C) b2 Agonist milial dyslipidemia. Recent laboratory studies


(D) Histamine1 agonist show an elevated triglyceride level but normal


(E) Histamine2 agonist LDL and HDL cholesterol levels. Which of


the following explains the pathophysiology of
22. An 8-month-old boy is brought to the pediatri- this disease?


cian by his parents because he has recently lost
(A) Apolipoprotein E deficiency
the ability to crawl or hold his toys. On exami-


(B) LDL cholesterol receptor deficiency
nation the patient is tachypneic and breathing


(C) Lipoprotein lipase deficiency
with considerable effort; the liver is palpable


(D) VLDL cholesterol clearance deficiency
five finger widths below the right costal mar-


(E) VLDL cholesterol overproduction


Hig -Yield Princi les
24 Section I: General Principles • Questions
p



25. A 59-year-old woman with history of morbid cheeks. He has arachnodactyly, pes cavus,


obesity, hypercholesterolemia, and diabetes and bilaterally dislocated lenses, and demon-
mellitus presents to the emergency depart- strates developmental delay with mild mental
ment with complaints of substernal chest pain retardation. His mother is told that her child
lasting two hours. An ECG reveals ST eleva- might benefit from folic acid supplementation.
h
tions in the lateral leads. The troponin level Which is the most appropriate test to confirm
at admission is extremely elevated, and a cre- the diagnosis?
atine kinase-myocardial bound test is pend-
(A) Enzymatic assay for the enzyme HGPRT
ing. Which of the following is a key cell me-



(B) Genetic studies demonstrating a mutation
diator in the pathogenesis of an atherosclerotic



in type I collagen
plaque?
(C) Genetic studies indicating >200 copies of



(A) γ-Interferon the CGG trinucleotide repeat on the X


(B) Complement chromosome


(C) Interleukin-6 (D) Nitroblue tetrazolium test
Biochemistry





(D) Natural killer cells (E) Nitroprusside cyanide test





(E) Platelet-derived growth factor


28. A 6-year-old boy is brought to his pediatrician’s


26. A 53-year-old man presents to his physician, office by his parents, who report that the child


because he has blood in his urine and some has been unusually thirsty for the past week.
low back pain. A gross specimen of kidneys He also has increased urinary frequency and
from a patient with the same condition is has wet the bed three times in the past two
shown in the image. Which of the following weeks. A random blood glucose level is 215
also is associated with this disorder? mg/dL. The pediatrician suspects that the
child has type 1 diabetes mellitus caused by
autoimmune destruction of insulin-producing
pancreatic b cells. Which of the following is
the transporter for glucose to enter pancreatic
b cells?
(A) GLUT 1


(B) GLUT 2


(C) GLUT 4


(D) Simple diffusion


29. Hemoglobin consists of four polypeptide sub-


units: two a subunits and two b subunits. The
arrangement of these subunits shifts between
a taut and relaxed conformation, resulting in
Reproduced, with permission, from USMLERx.com. changes in hemoglobin’s oxygen affinity. At a
given partial pressure of oxygen, which of the
following will decrease hemoglobin’s affinity
(A) Astrocytomas
for oxygen?


(B) Berry aneurysm


(C) Ectopic lens (A) Decreasing the partial pressure of carbon




(D) Optic nerve degeneration dioxide


(E) Squamous cell carcinoma (B) Increasing the amount of 2,3-bisphospho-




glycerate in RBCs
27. At a routine check-up, a 7-year-old boy is (C) Increasing the number of oxygen mole-




found to have osteoporosis. The patient is tall cules bound to a hemoglobin from one to
and thin with pale skin, fair hair, and flushed three
Hig -Yield Princi les
Chapter 2: Biochemistry • Questions 25

h




(D) Increasing the pH by moving from periph- (A) Adenosine deaminase




eral tissue to lung (B) Homogentisic acid oxidase



(E) The presence of excess carbon monoxide (C) Lysosomal a-1,4-glucosidase




(D) Ornithine transcarbamylase



30. A 15-year-old boy presents with prolonged fa- (E) Porphobilinogen deaminase

p




tigue and mild jaundice following a serious
infection. Blood tests reveal hemoglobin of 33. A 2-year-old boy is brought by his parents to



10.5 g/dL and an elevated reticulocyte count. the emergency department after the discovery
A peripheral blood smear reveals Heinz bodies. of blood in a wet diaper. The physician pal-
Which of the following best describes the nor- pates an abdominal mass in the right flank. CT
mal action associated with this patient’s meta- of the abdomen reveals a large tumor invading
bolic defect? the right kidney; the gross specimen is shown
in the image. Cytogenetic analysis of the tu-
(A) To generate glucose-6-phosphate in all
mor cells reveals a deletion of chromosome


cells

Biochemistry
11p. Which of the following is the most likely
(B) To generate glucose-6-phosphate in RBCs
diagnosis?


only
(C) To generate mucopolysaccharides


(D) To regenerate reduced nicotinamide ade-


nine dinucleotide phosphate in all cells
(E) To regenerate reduced nicotinamide ade-


nine dinucleotide phosphate in RBCs only

31. Hyperparathyroidism is a common manifesta-




tion of several distinct genetic disorders that
predispose to endocrine gland neoplasia and
cause hormone excess syndromes. Which of
the following is a consequence of parathyroid
hormone?
(A) Increase calcium absorption in the small


intestine
(B) Inhibit the production of 1,25-dihydroxyvi-


tamin D
(C) Promote calcium excretion in the renal tu-


bules
(D) Stimulate further secretion of parathyroid


hormone
(E) Stimulate phosphate reabsorption in the


renal tubules

32. A 22-year-old woman presents to the hospital




with severe abdominal pain, abdominal dis- Reproduced, with permission, from USMLERx.com.
tention, and ileus, along with peripheral neu-
ropathy. Her boyfriend notes that she has been
(A) Adrenocortical adenoma
acting strange lately, and that she “seems like


(B) Neuroblastoma
a different person.” Which of the following en-


(C) Renal cell carcinoma
zymes is deficient in this patient?


(D) Transitional cell carcinoma


(E) Wilms tumor


Hig -Yield Princi les
26 Section I: General Principles • Questions
p



34. A 6-year-old boy is diagnosed with a worsening 37. A term child is delivered by spontaneous vagi-




ataxic gait and a cardiac dysrhythmia. His un- nal delivery without complications. Upon
cle also has this condition, but his symptoms physical examination the child has bilateral
did not appear until he was 12 years of age. hip dislocations, restricted movement in shoul-
What is the molecular mechanism of this dis- der and elbow joints, and coarse facial features.
h
ease? Laboratory studies show that the activities of
b-hexosaminidase, iduronate sulfatase, and
(A) Unstable repeats affect protein folding
arylsulfatase A are deficient in cultured fibro-


(B) Unstable repeats affect protein splicing
blasts, but are 20 times normal in the patient’s


(C) Unstable repeats cause an amino acid sub-
serum. The primary abnormality in this disor-


stitution
der is associated with which of the following
(D) Unstable repeats impede protein transla-
organelles?


tion
(E) Unstable repeats result in a truncated pro- (A) Golgi apparatus





tein (B) Lysosomes
Biochemistry



(C) Ribosomes



35. A 3-month-old child is brought to his pediatri- (D) Rough endoplasmic reticulum




cian’s office for a check-up. On examination, (E) Smooth endoplasmic reticulum


the physician notices that he has a social smile,
but does not hold his head up on his own or 38. Patients with albinism appear white-pink (skin


make noises. The infant also has pale skin, ec- color), have white hair, and have nonpig-
zema, odd odor, and hyperreflexia. What is the mented or blue irises. In many cases, these
most appropriate treatment for the condition individuals may have melanocytes, but lack
the infant most likely has? melanin in their skin. What is the most use-
ful advice to give to a guardian of a child diag-
(A) A diet low in isoleucine and leucine
nosed with albinism?


(B) A diet low in phenylalanine


(C) A diet low in tyrosine (A) To avoid foods with lactose




(D) A high-protein diet (B) To avoid foods with phenylalanine




(E) Recombinant enzyme therapy (C) To avoid strenuous activity




(D) To give growth hormone to help the child


36. A 63-year-old man who is an alcoholic is grow to a normal height


brought into the emergency department by his (E) To wear clothing and sunscreen that pro-


daughter. She states that the patient’s memory tect from the sun when outside
has been very poor, and he constantly creates
elaborate yet untrue stories. Physical examina- 39. A 65-year-old African-American man presents


tion reveals ataxia and bilateral horizontal nys- to his physician because of jaundice. He says
tagmus. Wernicke-Korsakoff syndrome, caused that in the past few months he has not had
by a water-soluble vitamin deficiency, is sus- much of an appetite and has lost 13.6 kg (30
pected. Which of the following conditions is lb). Physical examination is notable for a gall-
also a result of a water-soluble vitamin defi- bladder that is palpable. What set of character-
ciency? istics is expected in this patient?
(A) Increased erythrocyte hemolysis


(B) Neonatal hemorrhage


(C) Night blindness


(D) Osteomalacia


(E) Pellagra


Hig -Yield Princi les
Chapter 2: Biochemistry • Questions 27

h




Type of Urine Urine
Na+: 135 mEq/L
Choice
hyperbilirubinemia bilirubin urobilinogen K+: 4.5 mEq/L
conjugated normal
Cl-: 95 mEq/L
A
HCO3-: 9 mEq/L
conjugated
B
Glucose: 40 mEq/L

p
C unconjugated
Serum pH: 6.8
D unconjugated Lactate: 9.5 mmoL/L
E unconjugated
What metabolic process induced this patient’s
current condition?
Reproduced, with permission, from USMLERx.com.
(A) Decreased levels of glycerol 3-phosphate



(B) Elevated pyruvate levels



(A) A (C) Inappropriate induction of gluconeogene-




(B) B sis


Biochemistry
(C) C (D) Overproduction of reduced nicotinamide




(D) D adenine dinucleotide


(E) E (E) Thiamine deficiency




40. A 3-year-old boy recently developed weak- 42. A 78-year-old man with asthma presents to his




ness of his lower extremity and uses his arms primary care physician for an annual check-
to stand up even though his lower legs appear up. The physician performs a physical exami-
quite muscular. Laboratory tests reveal a cre- nation and orders routine blood work, which
atine kinase level of 20,000 U/L. A DNA test reveals a macrocytic anemia. Subsequent
confirms the working diagnosis. What is the laboratory tests show an elevated serum meth-
function of the altered gene product in this pa- ylmalonic acid level. A peripheral blood smear
tient? is shown in the image. If this patient’s vitamin
(A) Exocytosis of acetylcholine at the neuro- deficiency is not corrected, what neurological
symptoms is he most likely to experience?


muscular junction
(B) Linking actin filaments to laminin


(C) Promoting actin-myosin cross-bridge cy-


cling
(D) Receptor for acetylcholine


(E) Release of calcium from the sarcoplasmic


reticulum

41. A 55-year-old man is found unresponsive and




breathing rapidly in his apartment. His daugh-
ter found him while stopping by to visit. She
stated that she was concerned after he told her
on the phone that he was “drowning his sor-
rows,” having been fired from his job earlier
that day. He has no significant medical history Reproduced, with permission, from USMLERx.com.
other than moderate hypertension, for which
he takes a b-blocker. Relevant laboratory find-
(A) Confusion and confabulation
ings are:


(B) Deficiency in this vitamin does not cause


neurological symptoms
(C) Dysarthria and diplopia


(D) Paresthesias and ataxia


(E) Syncope and lethargy


Hig -Yield Princi les
28 Section I: General Principles • Questions
p



43. A young woman currently being treated for is notable only for the ocular findings shown


HIV is brought to the emergency department in the image. Deficiency of which enzyme is
because of a headache and cyanosis of her nail responsible for this disease?
beds and lips. She also reports feeling dizzy.
The resident on call immediately places her
h
on supplemental oxygen and draws blood for
arterial blood gas analysis. While drawing the
blood, he notes that the arterial blood has a
dark brown color. Blood gas analysis reveals a
pH of 7.39, partial oxygen pressure of 96 mm
Hg, partial carbon dioxide pressure of 35 mm
Hg, and oxygen saturation of 82% on room air.
What enzyme is primarily responsible for pre-
venting this condition in the normal adult?
Biochemistry

(A) ATPase


(B) Flavin adenine dinucleotide reductase


(C) GTPase Reproduced, with permission, from USMLERx.com.


(D) Lactase


(E) Nicotinamide adenine dinucleotide reduc-
(A) a-Galactosidase A


tase


(F) Pyruvate kinase (B) Arylsulfatase A


(C) Hexosaminidase A




44. A mass is felt in the groin of an infant girl dur- (D) Iduronate sulfatase


(E) Lysyl hydrolase


ing a physical examination. Surgical resection


shows that it is a testicle. The baby is diag-
nosed with testicular feminization syndrome. 46. A pediatrician examines a baby with a defi-


In this syndrome, androgens are produced but ciency in fructose metabolism. Upon adminis-
cells fail to respond to the steroid hormones tration of a fructose bolus, the child becomes
because they lack appropriate intracellular re- symptomatic and blood glucose levels begin to
ceptors. After binding intracellular receptors, decrease. Which of the following will also oc-
steroids regulate the rate of which of the fol- cur after the administration of a fructose bolus
lowing? in this patient?

(A) Initiation of protein synthesis (A) A fall in serum phosphate levels




(B) A rise in cellular ATP levels


(B) mRNA degradation


(C) A sustained rise in serum fructose levels


(C) mRNA processing


(D) An increase in the serum pH


(D) Protein translation


(E) Large amounts of fructose in the urine


(E) Transcription of genes




45. The wife in an Ashkenazi Jewish family brings


her 1-year-old daughter to the pediatrician.
Her previous pregnancy was uneventful and
resulted in a full-term healthy girl who is now
4 years old. Her younger daughter, however,
has demonstrated a progressive series of behav-
iors over the first year of life. Her motor skills
have diminished and she demonstrates an in-
creased startle reaction. Physical examination
Hig -Yield Princi les
Chapter 2: Biochemistry • Questions 29

h




47. A 28-year-old African-American man is re- 50. A 28-year-old woman is trying to conceive a




ceiving primaquine therapy for treatment of child. She has a nephew with fragile X syn-
malaria, which he contracted while visiting drome (a genetic disorder characterized by
Asia. He presents to his physician after noting trinucleotide repeat expansion) and she would
blood in his urine. Physical examination is sig- like to assess her risk as a carrier for the disease.

p
nificant for scleral icterus, and urinalysis shows Blood is drawn, and DNA is extracted and cut
hemoglobinuria. A peripheral blood smear by restriction enzymes that flank the CGG
shows spherocytes, bite cells, and Heinz bod- repeat region. The DNA is then treated with
ies. Which of the following is the most likely a labeled probe that binds the affected region
diagnosis? of the gene. The woman is found to carry one
normal X chromosome and one X chromo-
(A) Alkaptonuria
some with some expansion of the CGG se-


(B) Cystinuria
quence. However, the number of CGG repeats


(C) Glucose-6-phosphate dehydrogenase defi-
in this X chromosome is not sufficient to alter


ciency

Biochemistry
phenotype. Which lane on the Southern blot
(D) Hereditary fructose intolerance
represents this woman’s genotype?


(E) Hereditary spherocytosis


(F) Lactase deficiency


Affected Unaffected
48. A neonate born at 28 weeks’ gestation is having nephew husband A B C D E


difficulty breathing. On physical examination, _
the neonate’s heart rate is 120/min, blood pres-
sure is 100/60 mm Hg, and respiratory rate is
55/min. He has nasal flaring and subcostal re-
tractions. Which of the following components
+
is deficient in this infant?
(A) Dipalmitoyl phosphatidylcholine Reproduced, with permission, from USMLERx.com.


(B) Elastase


(C) Functional cilia
(A) A


(D) Phosphatidylglycerol


(B) B


(E) Sphingomyelin


(C) C




49. A group of scientists at a pharmaceutical com- (D) D


(E) E


pany are conducting in vitro experiments to in-


vestigate the effects of an antineoplastic drug.
Under the microscope, it appears that with
treatment, the majority of the cells are arrested
at a stage in which their chromosomes are
aligned in the vertical axis of the cells. Which
antineoplastic agent has a mechanism of ac-
tion similar to the one described?
(A) 5-Fluorouracil


(B) Cyclophosphamide


(C) Etoposide


(D) Methotrexate


(E) Vincristine


Hig -Yield Princi les
30 Section I: General Principles • Answers
p



An s w e r s

1. The correct answer is G. This patient has xe- Answer E is incorrect. Succinyl-CoA down-


roderma pigmentosa, an autosomal recessive regulates its own synthesis by inhibiting the
h
disease characterized by a defect in excision enzyme responsible for dehydrogenation of
repair. This disease results in an inability to a-ketoglutarate.
repair thymidine dimers that can form in the
presence of ultraviolet light. This can lead to 3. The correct answer is E. Characteristic symp-



the development of skin cancer and photosen- toms of hyperthyroidism include tachycardia,
sitivity. heat intolerance, weight loss, weakness, tremu-
lousness, and diarrhea. This patient also displays
Answer A is incorrect. Methylation of a partic-
another symptom of elevated thyroid hormone
ular gene does not cause xeroderma pigmento-
levels, exophthalmos. Thyroid hormone enters
sum.
Biochemistry

target cells through carrier-mediated transport


Answer B is incorrect. A missense mutation or possibly diffusion, and binds to nuclear re-
does not cause xeroderma pigmentosum. ceptors. The hormone-receptor complex then
binds DNA and acts as a transcription factor,
Answer C is incorrect. A nonsense mutation
regulating the transcription of genes. Tran-
does not cause xeroderma pigmentosum.
scription results in the production of RNA,
Answer D is incorrect. A mutation in the en- specifically messenger RNA.
hancer region of a gene does not cause xero-
Answer A is incorrect. Carbohydrate is pro-
derma pigmentosum.
duced by the joining of sugar molecules dur-
Answer E is incorrect. A mutation in the oper- ing cellular metabolism; for example, glycogen
ator region of a gene does not cause xeroderma is produced from the linkage of glucose mol-
pigmentosum. ecules by glycogen synthase.
Answer F is incorrect. A mutation in the pro- Answer B is incorrect. DNA binding occurs
moter region of a gene does not cause xero- when the thyroid hormone molecule enters
derma pigmentosum. the nucleus and binds on nuclear receptors.
However, this leads to the transcription of
2. The correct answer is B. Citrate, formed from RNA from DNA, not production of new DNA


oxaloacetate and acetyl CoA by the enzyme or DNA replication.
citrate synthase, inhibits phosphofructokinase
and allosterically activates acetyl CoA carbox- Answer C is incorrect. Fatty acid is synthe-
ylase. Citrate synthase regenerates a molecule sized from acetyl-coenzyme A (CoA) in the
of CoA and is an important regulator of the tri- cell cytoplasm via the action of acetyl-CoA car-
carboxylic acid cycle. It is inhibited by adeno boxylase and other enzymes.
­
sine triphosphate. Answer D is incorrect. Protein may be pro-
Answer A is incorrect. a-Ketoglutarate is not duced from the RNA template but is not the
an important regulator of the tricarboxylic acid first molecule produced in response to thyroid
cycle, but it is an important intermediate in hormone.
protein metabolism.
4. The correct answer is C. The patient has al-


Answer C is incorrect. Malate is not an impor- kaptonuria, a condition corresponding to the
tant regulator of the tricarboxylic acid cycle, one described in the stem. A deficiency of the
but it is important in the malate shuttle. enzyme homogentisic acid oxidase leads to de-
Answer D is incorrect. Oxaloacetate is not an position of homogentisic acid in the joints and
important regulator of the tricarboxylic acid cartilage, giving them a dark color (ochronosis)
cycle, but it is important in glyconeogenesis. and resulting in degenerative changes. Clas-
Hig -Yield Princi les
Chapter 2: Biochemistry • Answers 31

h




sically, the urine of these patients turns black sickle cell anemia. The b-A-globin gene re-
on contact with air or when the urine is made sults in a 1.15-kb fragment of DNA cut by the
alkaline. The associated defect is on chromo- MstII restriction enzyme. The b-S-globin gene
some 3. results in a 1.35-kb band because the single
base-pair mutation responsible for sickle cell
Answer A is incorrect. A deficiency in a-keto-

p
anemia eliminates an MstII restriction site. A
acid dehydrogenase causes maple syrup urine
heterozygote will have two bands indicating
disease, a metabolic disorder of autosomal re-
one normal allele with an intact MstII site
cessive inheritance that affects the metabo-
(two fragments), and a mutant allele with a
lism of branched-chain amino acids (leucine,
missing MstII site (one fragment).
isoleucine, and valine) and causes the urine
of affected patients to smell like maple syrup. Answer A is incorrect. The band in lane A is
The urine does not, however, turn black upon from a sickle cell anemia patient with two cop-
standing. The disease is not classically associ- ies of the b-S-globin gene. This gene results
ated with arthritis in middle-aged individuals. in a 1.35-kb band because the single base-pair

Biochemistry
mutation responsible for sickle cell anemia
Answer B is incorrect. A deficiency in galac-
eliminates an MstII restriction site.
tokinase causes galactosemia and galactosuria,
but is otherwise a fairly benign condition and Answer C is incorrect. The band in lane C
would not present with any of the symptoms is from an unaffected patient with two copies
seen in this patient. Other symptoms would of the b-A-globin gene. The gene results in a
be cataracts in affected children, owing to the 1.15-kb fragment of DNA cut by the MstII re-
accumulation of galactitol, a by-product of ga- striction enzyme.
lactose metabolism when galactokinase is not
Answer D is incorrect. The bands in lane D
present.
could not result from any patient. The labeled
Answer D is incorrect. Orotate phosphoribo DNA probe does not bind to the 0.2-kb DNA
­
syltransferase is an enzyme involved in pyrimi- fragment and therefore would not be visual-
dine synthesis. Deficiencies in this enzyme or ized on the Southern blot.
in orotidine 5′-monophosphate decarboxylase
Answer E is incorrect. The bands in lane E
(an enzyme involved in the same pathway
could not result from any patient. The labeled
and located on the same chromosome) cause
DNA probe does not bind to the 0.2-kb DNA
a very rare disorder called hereditary orotic ac-
fragment and therefore would not be visual-
iduria. Symptoms include poor growth, mega-
ized on the Southern blot.
loblastic anemia, and orate crystals in urine.
Treatment involves cystidine or uridine to by- Answer F is incorrect. The bands in lane F
pass this step in pyrimidine synthesis and also could not result from any patient. The labeled
to negatively downregulate orotic acid produc- DNA probe does not bind to the 0.2-kb DNA
tion. fragment and therefore would not be visual-
ized on the Southern blot. A heterozygote will
Answer E is incorrect. A congenital deficiency
have two bands indicating one normal allele
of phenylalanine hydroxylase causes phenyl-
with an intact MstII site (two fragments), and
ketonuria. This enzyme converts phenylala-
a mutant allele with a missing MstII site (one
nine to tyrosine, and a deficit of this enzyme
fragment).
leads to a deficiency of tyrosine and a build-up
of phenylketones in the urine. It is associated 6. The correct answer is B. This patient was
with mental retardation and with the presence


started on an 3-hydroxy-3-methylglutaryl co-
of phenylketones in the urine (which do not enzyme A (HMG CoA) reductase inhibi-
classically turn black upon standing). tor (statin), which prevents the conversion of
HMG CoA to mevalonic acid, the rate-limiting
5. The correct answer is B. Lane B represents
step in cholesterol biogenesis. HMG CoA is


the Southern blot of a heterozygous carrier of
Hig -Yield Princi les
32 Section I: General Principles • Answers
p



formed from three acetyl CoA molecules and the DNA more than a million times. Dideoxy-
is a precursor to sterols and ketone bodies. nucleotides are not used in PCR techniques.
Muscle pain or injury resembling myositis has
Answer B is incorrect. Allele-specific oligo-
been known to occur with statins. Although
nucleotide probes are short, labeled DNA
the mechanism is unknown, it may be related
h
sequences complementary to an allele of in-
to a decrease in muscle tissue synthesis of
terest. These probes can be used to detect the
ubiquinone, a coenzyme used in muscle cell
presence of disease-causing mutations.
­
metabolism.
Answer D is incorrect. In a Southern blot pro-
Answer A is incorrect. Another common side
cedure, DNA is separated with electrophoresis,
effect of statins is transient elevation of trans-
denatured, transferred to a filter, and hybrid-
aminases. Alanine aminotransferase and aspar-
ized with a labeled DNA probe. Regions on
tate aminotransferase elevations are usually
the filter that base-pair with the labeled DNA
seen within 12 weeks after the onset of therapy
probes can be identified when the filter is ex-
and may be persistent. No studies have demon-
Biochemistry

posed to film that is sensitive to the radiola-


strated an adverse affect of this transaminitis.
beled probe.
Hepatomegaly, however, is not a known side
effect. Answer E is incorrect. In a Northern blot
procedure, RNA is separated by electrophore-
Answer C is incorrect. The teratogenicity of
sis, denatured, and transferred to a filter. RNA
statins is unknown. Fetal toxicity has been
is hybridized to a labeled radioactive DNA
demonstrated at high enough concentrations
probe. The hybridized RNA/DNA strand is ra-
to adversely affect the mother, but the conse-
dioactive and visualized when the filter is ex-
quence of standard doses is unknown. Statins
posed to film.
have not been shown to induce spontaneous
abortions. Answer F is incorrect. In a Western blot pro-
cedure, protein is separated by electrophore-
Answer D is incorrect. Statins have also been
sis and labeled antibodies are used as a probe.
implicated as causes of irritability and depres-
This technique can be used to detect the exis-
sion, specifically in patients with major depres-
tence of an antibody to a particular protein.
sion, but this relationship has not been con-
firmed. Homicidality is not caused by statins. Answer G is incorrect. Enzyme-linked immu-
nosorbent assay (ELISA) is an immunologic
Answer E is incorrect. Seizures are not a com-
technique used to determine whether a par-
mon side effect of statin therapy.
ticular antibody is present in a patient’s blood.
Labeled antibodies are used to detect whether
7. The correct answer is C. Sequencing is a
the serum contains antibodies against a spe-


laboratory technique that uses dideoxynucleo-
cific antigen precoated on an ELISA plate.
tides to randomly terminate growing strands of
DNA. Gel electrophoresis is used to separate
8. The correct answer is B. This patient has
the varying lengths of DNA. The DNA se-


a1-antitrypsin deficiency, a genetic disease
quence can then be read based on the position
characterized by a deficiency in the serine
of the bands on the gel.
protease inhibitor a1-antitrypsin. This protein
Answer A is incorrect. Polymerase chain reac- normally functions to inhibit neutrophil elas-
tion (PCR) is a laboratory technique used to tase in the lung. When deficient, there is over-
produce many copies of a segment of DNA. In abundant activity of elastase, which destroys
the procedure, DNA is mixed with two specific elastin and collagen in the alveolar walls, pro-
primers, deoxynucleotides and a heat-stable gressing to emphysema. Most patients with
polymerase. The solution is heated to denature a1-antitrypsin deficiency are homozygous for
the DNA and then cooled to allow synthesis. the Z allele. Clinically, a1-antitrypsin defi-
Twenty cycles of heating and cooling amplify ciency can affect the lung, liver, and less com-
Hig -Yield Princi les
Chapter 2: Biochemistry • Answers 33

h




monly the skin. In the lung, the most com- radiographic findings are more consistent with
mon manifestation is early onset panacinar emphysema.
emphysema, which is more prominent at the
lung bases than apices. Slowly worsening dysp 9. The correct answer is A. Calcium is main-



­
nea is the most common symptom, although tained in high concentrations outside of the

p
patients may initially complain of cough, spu- cell and in discrete compartments within the
tum production, or wheezing. As in this case, cell (eg, in mitochondria). Free intracellu-
patients who present early complaining of lar calcium can activate several enzymes the
episodes of wheezing and productive cough cumulative effect of which is to induce sig-
may be told they have asthma. Although treat- nificant cell injury. A few important enzyme
ment for asthma may initially improve symp- classes include ATPases, which decrease the
toms, it does not slow the progression of the ATP supply; phospholipases, which decrease
disease. Her x-ray of the chest shows a pattern membrane stability; endonucleases, which in-
typical for this disease; hyperinflated lungs, a duce DNA damage; and several proteases, re-

Biochemistry
flattened diaphragm, and hyperlucent lungs sponsible for protein breakdown.
due to decreased lung markings (it is difficult Answer B is incorrect. Free radical generation
to see at this resolution due to the overlying is a common mechanism of cell injury, but
breast tissue, but we expect that the lung mark- calcium excess does not induce free radical
ings would be especially absent at the bases). generation.
a1-Antitrypsin deficiency can also cause cir-
rhosis of the liver and panniculitis of the skin. Answer C is incorrect. Activation of proteases
and phospholipases induces the breakdown of
Answer A is incorrect. a1-Antitrypsin defi- necessary components of cell membranes.
ciency is characterized by low levels of a prote-
ase inhibitor. This leads to elevated activity of Answer D is incorrect. ATP depletion, result-
the protease elastase and increased destruction ing from the activation of ATPases, can con-
of elastin. tribute to the inhibition of glycolysis.
Answer C is incorrect. Mutations in the tu- Answer E is incorrect. Inhibition of oxidative
mor suppressor gene p53 lead to uncontrolled phosphorylation is an effect of ATP depletion
cellular proliferation. Such mutations are caused by enzyme activation. Although this
commonly seen in the lung cancers associated may contribute to cell damage, it is not the
with smoking (small cell and squamous cell), best answer. The enzyme activation resulting
and they have been found in many non-small from calcium excess is the root cause of the
cell types. This patient’s history and x-ray find- cell damage, and thus would be the primary
ings do not suggest cancer. insult responsible for the majority of cell dam-
age.
Answer D is incorrect. Mutations in the gene
encoding the cystic fibrosis transmembrane 10. The correct answer is B. G-protein coupled
conductance regulator (CFTR) on chromo-


receptors exist in an equilibrium between their
some 7 lead to the disease cystic fibrosis (CF). active and inactive states that is dependent on
CF is a multisystem disease that affects the res whether ligand is present, and the affinity of the
­
piratory tract, digestive tract, sweat glands, and ligand for the receptor. When active, these re-
reproductive tract. This patient does not pre ceptors catalyze guanine-nucleotide exchange
­
sent with symptoms characteristic of CF. (GTP for guanosine diphosphate) of their asso-
Answer E is incorrect. Airway inflamma- ciated G proteins. The Michaelis-Menten con-
tion, airway obstruction, and bronchial hyper- stant (Km) for any enzyme-catalyzed reaction
responsiveness are characteristic of asthma. is inversely proportional to the affinity of the
While this patient’s presentation mimicked enzyme for its substrate. Therefore, the Km for
that of asthma, her history, physical exam, and compound A will be lower than that for com-
pound B because compound A has a higher
Hig -Yield Princi les
34 Section I: General Principles • Answers
p



affinity for the receptor than compound B. porphobilinogen. Lack of this enzyme causes
The maximum rate of reaction (Vmax) will be porphobilinogen and δ-aminolevulinic acid to
reached at a lower concentration of A than it accumulate in the urine. Patients with acute
would for B, although the Vmax is unchanged. intermittent porphyria are not photosensitive,
but they do experience symptoms of painful
h
Answer A is incorrect. The Km of compound
abdomen, polyneuropathy, and psychological
A will be lower than that of compound B.
disturbances. They also have pink coloration of
Answer C is incorrect. Given that compounds their urine.
A and B have different affinities for the recep-
Answer D is incorrect. Ferrochelatase and
tor, their Michaelis-Menten constant values
δ-aminolevulinic acid (ALA) dehydrase are
cannot be the same.
sensitive to inhibition by lead. Thus, lead poi-
Answer D is incorrect. Vmax is directly propor- soning leads to an accumulation of copropor-
tional to the enzyme concentration, and is un- phyrin and ALA in the urine. Lead poisoning
affected by the concentration of substrates or is a problem seen in children who live in old
Biochemistry

competitive inhibitors. houses with chipped paint (lead was used in


Answer E is incorrect. Vmax is directly propor- paint manufacturing until the 1970s). Inges-
tional to the enzyme concentration, and is un- tion of large quantities of lead can cause lines
affected by the concentration of substrates or on the gingiva and epiphyses of long bones,
competitive inhibitors. encephalopathy, erythrocyte basophilic stip-
pling, abdominal colic, sideroblastic ane-
11. The correct answer is C. This individual suf- mia, and neuropathy leading to foot and wrist
drops. It is not associated with photosensitivity.


fers from porphyria cutanea tarda. The por-
phyrias are diseases resulting from enzymatic The first line of treatment is dimercaprol and
deficiencies in heme biosynthesis, and por- EDTA to bind up the free lead in the serum.
phyria cutanea tarda is the most common Answer E is incorrect. Porphobilinogen deam-
form. This disorder is caused by deficiency of inase deficiency, not excess, results in acute in-
uroporphyrinogen decarboxylase, the hepatic termittent porphyria. One would expect to find
enzyme that catalyzes the formation of copro- δ-aminolevulinic acid and porphobilinogen in
porphyrinogen III from uroporphyrinogen III. the urine and no photosensitivity.
Lack of this enzyme results in uroporphyrin
accumulation in the urine (giving the urine 12. The correct answer is B. This woman suffers


a tea-colored appearance) and uroporphy- from b-thalassemia major, the most severe
rinogen accumulation systemically. This com- form of b-thalassemia, in which the b-chain is
pound absorbs light and releases heat, causing absent. Clinically b-thalassemia major mani-
extreme photosensitivity. fests as severe hemolysis and ineffective eryth-
Answer A is incorrect. Hereditary copropor- ropoiesis. These individuals are transfusion
phyria is a disease due to the deficiency of dependent and frequently develop iron over-
coproporphyrinogen oxidase, the enzyme that load. The consequences of iron overload due
catalyzes the formation of protoporphyrinogen to transfusion dependency or secondary hemo-
from coproporphyrinogen III. Coproporphy- chromatosis are described in the stem. These
rinogen III accumulates in the urine. Patients manifestations are due to iron deposition in
with this condition tend to be photosensitive. various tissues including the pancreas, heart,
and skin. b-Thalassemia is more common
Answer B is incorrect. Acute intermittent por- among Mediterranean populations, whereas
phyria is caused by a deficiency in porphobi- b-thalassemia is more common among Asian
linogen deaminase (also called uroporphyrino- and African populations.
gen 1 synthetase), the enzyme that catalyzes
the formation of pre-uroporphyrinogen from Answer A is incorrect. This answer describes
the most severe form of a-thalassemia, a disease
Hig -Yield Princi les
Chapter 2: Biochemistry • Answers 35

h




in which a fetus is unable to make any func- drug is currently approved to inhibit aldose re-
tional hemoglobin aside from the γ4-tetramer ductase, but aldose reductase inhibitors such
(Hb Bart). Clinically a-thalassemia manifests as epalrestat and ranirestat are currently being
as congestive heart failure, anasarca, and intra tested.

­
uterine fetal death.
Answer A is incorrect. 3-Hydroxy-3-methyl-

p
Answer C is incorrect. This answer describes glutaryl coenzyme A (HMG CoA) reductase
hereditary hemochromatosis, a disease caused catalyzes the conversion of HMG CoA into
by iron overload due to an intrinsic defect in mevalonate and eventually into cholesterol. In-
the body’s ability to control the absorption of hibition of this enzyme is commonly affected
iron. Clinically this disease manifests in a man- by statin drugs to reduce cholesterol levels, but
ner similar to that of secondary hemochroma- it would not help prevent the development of
tosis. However, the laboratory picture in hered- cataracts.
itary hemochromatosis is not characterized by
Answer B is incorrect. Adenosine deaminase
hemolysis.

Biochemistry
inhibition would result in problems in the pu-
Answer D is incorrect. This answer describes rine salvage pathway. Disrupting this pathway
hereditary spherocytosis, a disease in which would result in excess ATP and dATP via feed-
mutations in either the ankyrin or spectrin back inhibition of ribonucleotide reductase.
gene contribute to instability of the RBC This excess ATP prevents DNA synthesis and
plasma membranes. This condition is charac- thus affects lymphocyte development. Congen-
terized by extravascular hemolysis. Clinically ital deficiency of this enzyme results in severe
this disease manifests as gallstones, anemia, combined immunodeficiency. Inhibition of
jaundice, and splenomegaly. The definitive this enzyme would not prevent the develop-
treatment is splenectomy, thus obviating any ment of cataracts.
need for chronic blood transfusion.
Answer D is incorrect. Galactose-1-phosphate
Answer E is incorrect. This answer describes (G-1-P) uridyltransferase is important in the
Wilson disease, a disease in which failure of breakdown of galactose; it catalyzes the for-
copper to enter the circulation in the form of mation of glucose-1-phosphate from G-1-P.
ceruloplasmin results in copper accumulation Hereditary deficiency of this enzyme leads to
in the liver, brain, and cornea. Clinically this hepatosplenomegaly, mental retardation, jaun-
disease manifests as parkinsonian symptoms, dice, and cataract formation. Inhibition of this
Kayser-Fleischer rings, asterixis, and dementia. enzyme in an adult would certainly not pre-
vent the development of cataracts.
13. The correct answer is C. Aldose reductase
Answer E is incorrect. Hexokinase is the en-


catalyzes the breakdown of glucose into sorbi-
zyme that catalyzes the first step in the catab-
tol. Sorbitol is then metabolized to fructose, a
olism of glucose, converting glucose to glu-
process that is relatively slow. In patients with
cose-6-phosphate. It is stimulated by insulin.
hyperglycemia, as would be present in this pa-
Inhibition of hexokinase would not prevent the
tient with poorly controlled diabetes, sorbitol
development of cataracts in this patient. Con-
accumulation with the cells of the lens leads
genital hexokinase deficiency is a rare autoso-
to a rise in intracellular osmolality, causing
mal recessive condition that results in severe
water movement into the cells. This results in
hemolysis. Inhibition of hexokinase would
cellular swelling and osmotic damage. It also
likely have a similar, albeit less severe, result.
leads to a decrease in intracellular myoinositol,
interfering with cellular metabolism. Swelling Answer F is incorrect. Insulin-like growth fac-
of lens fiber cells can lead to rupture and cata- tor (IGF) is a product synthesized in the liver
ract formation. Inhibition of aldose reductase that mediates many of the physiologic effects
could decrease sorbitol accumulation in the of growth hormone (GH). Its name refers to a
lens and thus prevent cataract formation. No high degree of structural similarity to insulin,
Hig -Yield Princi les
36 Section I: General Principles • Answers
p



and it is even capable of binding to the insu- Answer G is incorrect. Tyrosine kinase prod-
lin receptor directly, although with lower affin- ucts play a role in cell signaling through phos-
ity than insulin. Its effects include increased phorylation. The p53 gene product is not a ty-
protein synthesis, and IGF levels are especially rosine kinase.
high during puberty. Inhibition of IGF would
h
not help prevent the development of cataracts. 15. The correct answer is D. This question de-



scribes the polymerase chain reaction (PCR).
14. The correct answer is F. The p53 gene protein PCR is a laboratory technique used to produce


product is a transcription factor that regulates many copies of a segment of DNA. In the pro-
apoptosis. It acts as a cell-cycle regulator, pre- cedure, DNA is mixed with two specific prim-
venting cells from undergoing division. Muta- ers, deoxynucleotides and a heat-stable poly-
tions in p53 cause uncontrolled cell division, merase. The solution is heated to denature
leading to various types of tumors. Another the DNA and then cooled to allow synthesis.
example of a cell-cycle regulator is the retino- Twenty cycles of heating and cooling amplify
Biochemistry

blastoma gene. the DNA over a million times.


Answer A is incorrect. The p53 gene product Answer A is incorrect. Enzyme-linked immu-
is involved in apoptosis induced by DNA dam- nosorbent assay (ELISA) is an immunologic
age and other stimuli, but it is not a caspase technique used to determine whether a par-
protein. ticular antibody is present in a patient’s blood.
Labeled antibodies are used to detect whether
Answer B is incorrect. p53 is involved in cell
the serum contains antibodies against a spe-
cycle regulation but not direct DNA repair ac-
cific antigen precoated on an ELISA plate.
tivity. DNA repair products are produced by
This is not the technique described above.
genes such as BRCA1 (chromosome 17) and
BRCA2 (chromosome 13), among others. Answer B is incorrect. Gel electrophoresis
uses an electric field to separate molecules
Answer C is incorrect. Membrane cell adhe-
based on their sizes.
sion products are produced by the APC gene
found on chromosome 5. Mutations in the Answer C is incorrect. Northern blots are sim-
APC gene lead to colon cancer. ilar to Southern blots except that in Northern
blotting, mRNA is separated by electrophoresis
Answer D is incorrect. p53 is not a G-protein
instead of DNA. This is not the technique de-
product. G proteins such as ras can be in-
scribed above.
volved in oncogenesis. Mutations in ras can
lead to cancer in the lungs, pancreas, and co- Answer E is incorrect. Sequencing is a labo-
lon as well as leukemia. ratory technique that utilizes dideoxynucleo-
tides to randomly terminate growing strands of
Answer E is incorrect. DNA polymerase is un-
DNA. Gel electrophoresis is used to separate
able to replicate at the end of chromosomes
the varying lengths of DNA. The DNA se-
(telomeres), resulting in the loss of DNA with
quence can then be read based on the position
each replication cycle. Telomerase is an en-
of the bands on the gel. This is not the tech-
zyme that adds repeats onto the 3′ends of chro-
nique described above.
mosomes to protect them from being recog-
nized as broken or damaged DNA. Although Answer F is incorrect. In a Southern blot pro-
most normal somatic cells do not express cedure, DNA is separated with electrophoresis,
enough telomerase to prevent telomerase attri- denatured, transferred to a filter, and hybrid-
tion with each cell division, telomerase is often ized with a labeled DNA probe. Regions on
reexpressed in cancer cells. However, p53 is the filter that base-pair with the labeled DNA
not involved in the process of adding nucleo- probes can be identified when the filter is
tides to telomeres. exposed to film that is sensitive to the radio-
Hig -Yield Princi les
Chapter 2: Biochemistry • Answers 37

h




labeled probe. This is not the technique de- enzymes that convert the D-ala D-ala peptide
scribed above. bridge to D-ala D-lac, preventing vancomycin
from binding. Vancomycin resistance is much
Answer G is incorrect. In a Western blot pro-
more common with Enterococcus faecium than
cedure, protein is separated by electrophore-
with Enterococcus faecalis. High-dose ampicil-
sis and labeled antibodies are used as a probe.

p
lin, often in combination with gentamicin, is
This technique can be used to detect the exis-
generally first-line treatment in urinary tract
tence of an antibody to a particular protein.
infections due to vancomycin-resistant Entero-
16. The correct answer is B. This woman has coccus.


symptoms of Hashimoto thyroiditis, an auto- Answer A is incorrect. Microorganisms be-
immune disorder resulting in hypothyroidism come resistant to quinolones through the al-
(also known as myxedema), although there teration of their gyrase.
may be a transient hyperthyroidism at the very
Answer B is incorrect. Microorganisms be-
onset of disease when follicular rupture occurs.

Biochemistry
come resistant to macrolides through the
It is a type IV hypersensitivity associated with
methylation of its rRNA at a ribosome-binding
autoantibodies to thyroglobulin, thyroid per-
site.
oxidase, and the thyroid-stimulating hormone
receptor itself. The most common presenting Answer C is incorrect. b-Lactamases are en-
symptoms of Hashimoto thyroiditis are those zymes produced by microorganisms that cleave
seen in this patient, as well as constipation b-lactam antibiotics, deactivating them. To
and dry skin. Histologic characteristics include overcome resistance, b-lactams are usually
massive infiltrates of lymphocytes with germi- given with b-lactamase inhibitors such as cla-
nal cell formation. Hashimoto thyroiditis is as- vulanic acid, tazobactam, and sulbactam.
sociated with the DR5 human leukocyte anti-
Answer E is incorrect. b-Lactam antibiotics
gen subtype, as is pernicious anemia, a disease
bind to penicillin-binding proteins (enzymes
that leads to vitamin B12 deficiency caused by
that synthesize peptidoglycan, a major com-
atrophic gastritis and destruction of parietal
ponent of bacterial cell walls), preventing cell
cells.
wall synthesis by the microorganism. Microor-
Answer A is incorrect. Multiple sclerosis is as- ganisms such as methicillin-resistant Staphy-
sociated with the DR2 human leukocyte an- lococcus aureus and penicillin-resistant Strep-
tigen subtype. It is not associated with Hashi- tococcus pneumoniae have alterations in their
moto thyroiditis. penicillin-binding proteins that result in low
affinity and thus resistance to these b-lactams.
Answer C is incorrect. Psoriasis is associated
b-Lactam antibiotics include penicillins, ceph-
with the B27 human leukocyte antigen sub-
alosporins, monobactams, and carbapenems
type. It is not associated with Hashimoto thy-
(not vancomycin).
roiditis.
Answer D is incorrect. Steroid-responsive ne- 18. The correct answer is C. Kartagener syn-


phrotic syndrome is associated with the DR7 drome, or immotile cilia, is caused by a defect
human leukocyte antigen subtype. It is not as- in dynein that prevents effective movement of
sociated with Hashimoto thyroiditis. cilia. The full syndrome is characterized by si-
nusitis, bronchiectasis, situs inversus, and male
17. The correct answer is D. Vancomycin is a gly- infertility. Cilia play an important role in mov-


copeptide antibiotic that is effective in fighting ing mucus along the airway and clearing de-
only gram-positive bacteria. It binds tightly to bris; the absence of this function contributes to
a cell wall precursor that contains the termi- the pulmonary findings of the syndrome. Cilia
nal amino acid sequence D-ala D-ala and pre- are also very important for leukocyte move-
vents cell wall synthesis. Resistance to vanco- ment and phagocytosis. Infertility is present in
mycin is transferred via plasmids and encodes most patients due to immotile cilia.
5MEDBOOK.COM

NO WATERMARK
MEDICAL BOOK
EASY TO BUY

ONLY $5

ONLY $5

Amazon.com http://www.amazo...

Find your book you want Fill in Book URL Receive Free 55 first pages
in any site within 5 hours

5MEDBOOK.COM

Вам также может понравиться